GI 2 Kaplan Questions w/ explanations Revised

अब Quizwiz के साथ अपने होमवर्क और परीक्षाओं को एस करें!

A patient is admitted to the hospital with abdominal complaints. The patient undergoes a barium enema as part of the evaluation, and the findings on this study reveal "thumbprinting" of the colonic wall. Which of the following is the most likely diagnosis? A. Acute mesenteric ischemia B. Acute diverticulosis C. Acute colonic perforation D. Acute bowel obstruction E. Intussusception

A. Acute mesenteric ischemia Mesenteric ischemia causes thumbprinting on barium enema from the edematous mucosal folds.

64. A 7-month-old girl is brought to the office by her grandmother who is concerned that her granddaughter isn't gaining any weight. The grandmother lives out of state and occasionally visits her daughter. She was last here 3 months ago, when the girl was 4 months old; at the time the girl weighed 8 pounds. She was smiling and waving her arms and legs joyously when she was attended to. However, when the grandmother arrived a few days ago, she noticed at once that the child was nervous and often crying, and her weight was only 8.5 pounds. She had also noticed that the infant was somewhat unkempt, with long fingernails and a diaper rash that was very sore and looked infected in places. Her daughter, the infant's mother, had started working again and was having a hard time coordinating the infant's care, so she often had to leave her unattended for a couple of hours until her boyfriend could come and babysit a while. On physical examination, the infant indeed weighs 8.5 pounds. There is diffuse erythema and edema of the diaper area with areas of honey-colored crust-scale that spares the inguinal folds and does not involve skin outside of the occluded area. Her occipital skull is flattened and there is a circular patch of alopecia at the center of the flattened zone. She has poor muscle tone and cannot sit up appropriately. Which of the following is the most appropriate next step in management? A. Hospitalization for documentation of caloric intake and weight gain B. Prescription for oral antibiotics C. Reassurance D. Skin-prick testing E. A sweat chloride test

A. Hospitalization for documentation of caloric intake and weight gain The correct answer is A. The patient shows failure to thrive, along with signs and symptoms of abuse by deprivation: inadequate weight gain, unkempt look, impetigo-like irritant contact dermatitis in the diaper area, flattened occiput with pressure alopecia secondary to long hours of lying on the back, irritability, and poor muscle tone. Failure to thrive is failure to gain weight or deceleration of weight growth. Patients may show little subcutaneous fat, muscle wasting, rashes, poor tone, and weak cry. Hospitalization for documentation of caloric intake is sometimes necessary, especially in situations in which abuse by starvation or deprivation is suspected. Complete blood counts, urinalysis, liver function tests, and serum protein should be drawn. A sweat chloride test should be considered.

A 23-year-old professional basketball player presents to his health care provider 3 hours before game time complaining of abdominal pain. The symptoms began approximately 8 hours ago in a diffuse fashion. Two hours later, he began feeling nauseated and vomited twice. Over the past 4 hours the abdominal pain has become more severe and well-localized in the right lower quadrant. His examination now reveals well-localized pain in the right lower quadrant inferolateral to the umbilicus. Which of the following is the most likely diagnosis? A. Acute obstruction of the appendiceal lumen by a fecalith B. Acute onset of ileocolitis C. Acute onset of ischemic colitis D. Acute Yersinia infection E. Obstruction of the ileocecal valve by a mass

A. Acute obstruction of the appendiceal lumen by a fecalith Acute appendicitis is one of the most common surgeries performed in the United States and the most common surgical emergency in every decade of life. Although it can occur at any age, many patients are teenagers and young adults. This patient's presentation is typical for acute appendicitis, with initially poorly localized pain followed by nausea and vomiting. In classic appendicitis the pain shifts to the right lower quadrant, where it becomes more localized in the area known as McBurney point. In most patients acute obstruction of the appendiceal orifice by a fecalith initiates the acute appendicitis.

For patients with right-sided colorectal cancer, which of the following signs or symptoms is most likely to initially occur? A. Anemia B. Hematochezia C. Distended bowel from obstruction D. Change in bowel habits E. Change in diameter of the stool

A. Anemia Right-sided lesions in the bowel cause melena, iron deficiency anemia, and weakness. The anemia can progress without interfering with bowel habits because the stool is very liquid when it is on the right side of the colon. Because liquid can pass through very narrow openings, no obstruction would result.

A patient is admitted to the hospital with bleeding caused by arteriovenous malformation. Which of the following cardiac abnormalities is associated with gastrointestinal (GI) bleeding due to an arteriovenous (AV) malformation? A. Aortic stenosis B. Aortic regurgitation C. Mitral stenosis D. Mitral regurgitation E. Mitral valve prolapse

A. Aortic stenosis Of people with bleeding AV malformations, 25% will have aortic stenosis. The association between chronic GI bleeding due to angiodysplasia and calcific aortic stenosis has been termed Heyde syndrome.

A patient is admitted to the hospital with signs and symptoms consistent with biliary colic. Ultrasound fails to identify any stones in the gallbladder. Hepatobiliary iminodiacetic acid (HIDA) scan and IV injection of cholecystokinin fail to show any contractility of the gallbladder. Which of the following is the cause for this presentation? A. Biliary dyskinesia B. Bile duct stricture C. Choledochal cysts D. Primary biliary cholangitis E. Primary sclerosing cholangitis

A. Biliary dyskinesia Biliary dyskinesia has impaired ejection fraction of the gallbladder after injection of cholecystokinin, which is listed as gallbladder ejection fraction on the HIDA scan with cholecystokinin challenge.

Normal-weight patients with sickle cell disease who have undergone multiple bouts of crisis are most likely to develop which of the following? A. Black pigmented gallstones B. Brown pigmented gallstones C. Yellow, cholesterol gallstones D. Hepatitis C E. Cirrhosis

A. Black pigmented gallstones Sickle cell patients with chronic hemolysis are most likely to develop black pigmented stones.

A 63-year-old man has had mid-abdominal colicky pain with nausea, anorexia, and vomiting for the past 48 hours. The patient has a history of deep venous thrombosis and had a pulmonary embolus in the distant past, but he is not currently on anticoagulants. His temperature is 37ºC (98.6ºF), blood pressure 95/60 mm Hg, and pulse 102/min. On physical examination he has a distended and tympanitic abdomen, but he does not have an "acute abdomen." Abdominal radiographs show multiple distended loops of small bowel and distention of the right colon up to the middle of the transverse colon. Stool is positive for occult blood. Because of his history, a diagnosis of possible mesenteric venous thrombosis is entertained. Which of the following is the most appropriate next diagnostic step? A. CT scan of the abdomen B. Doppler ultrasound C. Exploratory laparotomy D. Laparoscopy E. Upper gastrointestinal endoscopy

A. CT scan of the abdomen A CT scan of the abdomen (or MRI scan, which was not offered as a choice) has excellent diagnostic yield when looking for mesenteric venous thrombosis. CT scan with contrast or CT angiogram will show bowel wall thickening, bowel dilation, pneumatosis intestinalis, portal venous gas pattern, and occlusion of the mesenteric vasculature. The CT scan will also provide evidence showing the extent of bowel compromise from ischemia. Mesenteric angiography also remains a viable option in the evaluation of a patient suspected of having mesenteric thrombosis. The clinical picture is indeed very suggestive for that condition.

A 66-year-old man comes to the urgent care clinic with progressive jaundice, which he first noticed 6 weeks ago. He has never had any chronic medical conditions and takes no medications. He has smoked 1 pack of cigarettes a day for 30 years and drinks a glass of wine each evening. His total bilirubin is 22 mg/dL (normal <1.9 mg/dL), with a direct (conjugated) bilirubin 16 mg/dL (normal <0.3 mg/dL). Transaminases are minimally elevated, whereas his alkaline phosphatase is about 6 times the upper limit of normal. Sonogram shows dilated intrahepatic ducts, dilated extrahepatic ducts, and a very distended, thin-walled gallbladder without stones. Which of the following is the most appropriate next step in diagnosis? A. CT scan of the upper abdomen B. Endoscopic retrograde cholangiopancreatography (ERCP) C. Exploratory laparotomy D. Percutaneous transhepatic cholangiogram (PTC) E. Serologies to define the type of hepatitis

A. CT scan of the upper abdomen Obstructive jaundice is evident by the high alkaline phosphatase and the dilated biliary ducts. Malignancy is suggested by the dilated, thin-walled gallbladder. If there is a cancer of the head of the pancreas, CT scan has a good chance of showing it in a noninvasive manner.

Which of the following has been shown to decrease the risk for colorectal cancer formation? A. Calcium supplementation B. Treatment of hypertension with beta-blockers C. Treatment of clinical depression D. Supplementation with vitamin A E. Low-salt diet

A. Calcium supplementation Calcium supplementation and, possibly, daily aspirin therapy can decrease the risk for colon cancer.

Which of the following electrolyte abnormalities is most commonly seen with squamous cell carcinoma of the esophagus? A. Hypercalcemia B. Hypocalcemia C. Hyperkalemia D. Hypokalemia

A. Hypercalcemia Hypercalcemia occurs in the setting of adenocarcinoma of the esophagus because of production of parathyroid hormone-related peptide hypersecretion as part of this paraneoplastic syndrome.

A 19-year-old woman comes to the urgent care clinic with a 2-week history of frequent episodes of loose stools. The symptoms are accompanied by severe fecal urgency, and she has awoken with diarrhea several nights weekly. Over the past week, the stools have become increasingly bloody. A sigmoidoscopy reveals continuous, symmetric inflammation from the anal verge to the proximal sigmoid colon. Which of the following infections is most likely causing this patient's symptoms? A. Campylobacter jejuni B. Cryptosporidium C. Giardia lamblia D. Herpes simplex E. Yersinia enterocolitica

A. Campylobacter jejuni This patient has an acute onset of bloody diarrhea consistent with colitis, which is confirmed with the sigmoidoscopy. Of the organisms listed, Campylobacter jejuni is the one most likely to cause these symptoms. This organism can cause diarrhea in all age groups, although the peak of incidence is in young children. C. jejuni can be acquired through exposure to contaminated food (especially undercooked poultry) or water, or through exposure to infected domestic or wild animals. The diarrhea can be either watery or bloody and is often accompanied by a sometimes high fever. White blood cells are commonly seen in the fecal material. You should also be aware that there appears to be an association between summer outbreaks of diarrheal disease caused by this organism and subsequent development of the peripheral nervous system autoimmune disease Guillain-Barré syndrome.

A patient in the ICU is evaluated for abdominal pain and is diagnosed with acalculous cholecystitis. Which of the following treatments should be pursued? A. Cholecystectomy B. Replacement of thyroid hormone C. Broad spectrum antibiotics D. Endoscopic retrograde cholangiopancreatography (ERCP) E. Magnetic resonance cholangiopancreatography (MRCP)

A. Cholecystectomy The risk for development of acalculous cholecystitis is significant illness, and emergency cholecystectomy or percutaneous drainage of the gallbladder is recommended for treatment of these patients. These patients have a high mortality so prompt intervention is needed.

A 42-year-old obese woman experiences episodic abdominal pain. She notes that the pain increases after the ingestion of a fatty meal. Action of which of the following hormones is responsible for the postprandial intensification of her symptoms? A. Cholecystokinin (CCK) B. Gastrin C. Pepsin D. Secretin E. Somatostatin

A. Cholecystokinin (CCK) This woman has a risk profile (female, fat, forties) and symptomatology consistent with gallstones (cholelithiasis). As would be expected, contraction of the gallbladder following a fatty meal often exacerbates the pain caused by gallstones. Cholecystokinin (CCK) is the hormone responsible for stimulation of gallbladder contraction; the release of CCK is stimulated by dietary fat. It is produced in I cells of the duodenum and jejunum. In addition to gallbladder contraction, CCK also stimulates pancreatic enzyme secretion and decreases the rate of gastric emptying.

A patient has long-standing Crohn disease. As the disease progresses, several complications can occur. Which of the following complications is most likely to occur with this condition? A. Cholesterol gallstones B. Toxic megacolon C. Eosinophilic colitis D. Sarcoidosis E. Polymyalgia rheumatica

A. Cholesterol gallstones Crohn disease can be complicated by cholesterol gallstones because of bile salt malabsorption. Other complications that can occur with Crohn disease include calcium oxalate kidney stones and amyloidosis.

A patient with long-standing inflammatory bowel disease is diagnosed with primary sclerosing cholangitis. Ongoing symptoms occur with this condition. Which of the following treatments is indicated for managing the pruritus that occurs with this condition? A. Cholestyramine (Questran) B. Supplemental vitamin D and calcium C. Lactulose D. Antihistamines such as diphenhydramine (Benadryl) E. Glucocorticoids such as prednisone

A. Cholestyramine (Questran) Cholestyramine helps the itching that is seen with primary sclerosing cholangitis.

You see a patient who is being evaluated for gastrointestinal-related complaints thought to be due to Zollinger-Ellison (Z-E) syndrome. Patients with this condition will have complaints related to this syndrome that include all of the following signs or symptoms EXCEPT: A. Complaints of early satiety B. Symptoms refractory to therapy C. Ulcers being found in atypical locations D. Elevation of serum gastrin level E. Complaints of diarrhea

A. Complaints of early satiety Patients with Z-E syndrome have involvement in pancreatic or duodenal locations, so this condition should not cause early satiety.

146. Dark urine and pale or clay-colored stools occur in which of the following settings? A. Elevated conjugated bilirubin B. Elevated unconjugated bilirubin C. Esophageal variceal bleeding D. Hepatic adenoma E. Cavernous hemangioma

A. Elevated conjugated bilirubin The correct answer is A. Elevated conjugated bilirubin results in dark urine and pale-colored stools, as the conjugated bilirubin cannot get into the gastrointestinal tract, most commonly because of obstruction.

Which of the following is known to precipitate hepatic encephalopathy in the setting of liver failure? A. Gastrointestinal bleeding B. Diarrhea C. Lactulose D. High-carbohydrate meal E. Asthma flare

A. Gastrointestinal bleeding The typical precipitants of hepatic encephalopathy in the setting of a patient with underlying liver disease include gastrointestinal bleeding, azotemia, constipation, high-protein meals, hypokalemic alkalosis, CNS depressant medications, hypoxia, hypercarbia, and sepsis.

Black pigmented gallstones are associated with which of the following conditions? A. Hemolysis B. Infection C. Obesity D. Ileal resection E. Crohn disease

A. Hemolysis Black pigmented stones seen with chronic hemolytic conditions such as sickle cell disease.

A patient has long-standing hemochromatosis with resulting cirrhosis. Which of the following is the most likely complication of the cirrhosis? A. Hepatocellular cancer B. Hepatitis A C. Hepatitis B D. Hepatitis D E. Portal hypotension

A. Hepatocellular cancer Hemochromatosis leading to cirrhosis increases risk for hepatocellular cancer 200-fold.

Which of the following electrolyte disturbances is most likely to occur in the setting of acute, prolonged vomiting? A. Hypokalemia B. Hyponatremia C. Hypocalcemia D. Hypomagnesemia E. Metabolic acidosis

A. Hypokalemia Prolonged vomiting causes the loss of potassium, hydrogen, and chloride ions, which results in hypokalemia and hypochloremia. The loss of these ions leads to hypokalemic, hypochloremic metabolic alkalosis.

Laboratory results consistent for hemochromatosis include all of the following EXCEPT: A. Increased total iron-binding capacity B. Elevated serum iron C. Elevated serum ferritin D. Elevated transferrin saturation E. Positive chromosomal abnormalities (HFE gene)

A. Increased total iron-binding capacity Hemochromatosis will decrease total iron-binding capacity because ample iron is available from the increased absorption and decreased elimination of iron from the body.

103. A patient is admitted to the hospital with the diagnosis of pyogenic liver abscess. Which of the following is the most common reason for this condition to develop? A. Infectious biliary tract obstruction B. Tapeworm infestation C. Schistosomiasis D. Amebiasis

A. Infectious biliary tract obstruction The correct answer is A. Biliary tract infection is the most common cause of pyogenic liver abscess.

A 32-year-old man has a 15-year history of celiac sprue. He admits to being noncompliant with the diet prescribed over the past 6 months and has lost 15 pounds during that time in association with frequent diarrhea. The patient's complete blood count reveals a depressed hemoglobin and hematocrit. Serum ferritin value is 15 μg/L (normal 20-250 μg/L). Which of the following is most likely to occur in this patient? A. Iron deficiency will produce a microcytic anemia B. Vitamin A deficiency will produce nystagmus C. Vitamin B12 deficiency will produce a megaloblastic anemia D. Vitamin D deficiency will produce hypercalcemia E. Vitamin K deficiency will produce hypokalemia

A. Iron deficiency will produce a microcytic anemia Celiac sprue will produce signs and symptoms of malabsorption of the proximal small bowel. Iron (along with folate and calcium) is preferentially absorbed in the proximal small bowel (mostly the duodenum) and is not well compensated for in the distal small intestine when there is proximal small bowel malabsorption. Vitamin A, D, and K deficiencies will occur but do not produce the described symptoms. Iron-deficiency anemia is characterized by a microcytic, hypochromic pattern on the peripheral blood smear. In addition, the serum ferritin level of less than 16 ug/L is strongly suggestive of a microcytic process. It should be remembered that the red blood cells become smaller and the ferritin levels fall before the fall in the hemoglobin and hematocrit.

A patient with gastric cancer has a palpable ovary that is thought to be secondary to metastatic disease to the ovary. What is this metastatic ovarian tumor known as? A. Krukenberg tumor B. Blumer shelf tumor C. Sister Mary Joseph nodule D. Virchow node E. Irish node

A. Krukenberg tumor Krukenberg tumor is gastric cancer that has metastasized to the ovary.

A patient is diagnosed with colorectal cancer located in the lower sigmoid colon. Which of the following sites is most likely to have metastatic spread? A. Liver B. Brain C. Kidney D. Adrenals E. Lungs

A. Liver The liver is the most likely site of spread of colorectal cancer.

A patient is seen in the office with complaints related to severe diarrhea. The patient has a high fever in addition to the diarrhea. The patient has just returned from a trip to the Bahamas. Which of the following should be avoided as part of the treatment regimen? A. Loperamide (Imodium) B. Gatorade mixed with water C. Ciprofloxacin (Cipro) D. Potassium, as indicated E. Rifaximin

A. Loperamide (Imodium) Antidiarrheal agents such as loperamide need to be avoided in the setting of traveler's diarrhea unless it is mild diarrhea without fever.

A postoperative patient is seen in the hospital during medicine rounds. The patient is complaining of abdominal pain. Abdominal plain film reveals a uniform distribution of gas in the small bowel, colon, and rectum. Which of the following electrolytes will need to be replaced to the greatest extent to clear this obstruction? A. Potassium B. Sodium C. Calcium D. Magnesium E. Chloride

A. Potassium Provide potassium in the setting of paralytic ileus. There is no back-up system in place for saving potassium that is being third-spaced in the setting of paralytic ileus.

A patient was recently hospitalized for sepsis and had multiple antibiotic regimens during his hospitalization. The patient read an article regarding pseudomembranous colitis and its high incidence for patients in the hospital and for patients who have taken antibiotics. Which of the following signs or symptoms would be consistent with pseudomembranous colitis? A. Profuse watery diarrhea B. Bloody diarrhea C. Diarrhea with a large amount of mucus D. Black diarrhea E. Green diarrhea

A. Profuse watery diarrhea Profuse watery diarrhea is consistent with Clostridium difficile colitis. This diarrhea is often foul-smelling, but it is typically not associated with bleeding.

A 40-year-old obese white woman, mother of 5 children, gives a history of repeated episodes of right upper quadrant abdominal pain. The pain is triggered with the ingestion of fatty foods and relieved with anticholinergic medications. The pain is spasmodic, radiates to the right shoulder and around to the back, and is accompanied by nausea and occasional vomiting. The patient is in no pain at the moment but is anxious to avoid further episodes. She is afebrile, and physical examination is unremarkable. Which of the following is the most appropriate next step in this patient's evaluation? A. Sonogram of the biliary tract and gallbladder B. Upper gastrointestinal series with barium C. Antibiotics, intravenous fluids, and nothing by mouth D. Endoscopic retrograde cholangiopancreatogram (ERCP) E. Laparoscopic cholecystectomy

A. Sonogram of the biliary tract and gallbladder The clinical description is classic for biliary colic caused by gallstones that are intermittently impacted at the cystic duct. The diagnostic study of choice to confirm the presence of gallstones is a sonogram. Confirmation that stones are present is needed prior to surgical cholecystectomy unless the patient has a gangrenous gallbladder.

Which of the following medications is the first-line treatment for primary biliary cholangitis? A. Ursodeoxycholic acid (Actigall) B. Clopidogrel (Plavix) C. Lactulose D. Pentoxifylline (Trental) E. Prednisone

A. Ursodeoxycholic acid (Actigall) Ursodeoxycholic acid improves biochemical and histology features of primary biliary cholangitis.

Which of the following is a known risk factor for the development of hepatic adenoma? A. Use of oral contraceptives B. Use of thiazide diuretics C. Smoking D. Use of vitamins containing iron E. Male sex

A. Use of oral contraceptives The use of oral contraceptives, being a woman, and anabolic steroid use are the main risk factors for hepatocellular adenomas.

Which of the following tumor markers is typically followed in the setting of pancreatic cancer? A. Carcinoembryonic antigen (CEA) B. CA 19-9 C. Thyroglobulin D. Alpha fetoprotein (AFP) E. Beta human chorionic gonadotropin (HCG)

B. CA 19-9 CA 19-9 is tumor marker for assessing for the return of pancreatic cancer.

The health care provider becomes concerned when after 30 hours, a newborn infant has not passed meconium. He was full term with a birth weight of 3,856 g (8 lb 8 oz). The pregnancy was uncomplicated. The infant appears well, with no respiratory distress. Slight abdominal distention is noted. Rectal examination reveals a slightly tight rectum and results in a greenish gush of stool. Which of the following tests will probably confirm the likely diagnosis? A. A stool culture B. A rectal biopsy C. A barium enema D. An alpha1-antitrypsin level E. A serum TSH level

B. A rectal biopsy Hirschsprung's disease or congenital aganglionic megacolon is caused by a congenital absence of the ganglion cells of both the Meissner and Auerbach plexuses. It is the most common cause of lower intestinal obstruction in the neonatal period. In early childhood it may present as chronic constipation with intermittent fecal soiling. It occurs predominantly in males and there is an increased family incidence. Surgical treatment is indicated, but the diagnosis is confirmed by a suction biopsy that can be easily performed without general anesthesia. The biopsy specimen would reveal an absence of ganglion cells in the submucosal and myenteric plexuses.

A 53-year-old man is admitted to the hospital for fever and abdominal pain. He has a history of cirrhosis and is known to be hepatitis C positive. He was diagnosed with cirrhosis 4 years ago. He denies any alcohol or tobacco use. His only medications are spironolactone (Alactone) and propranolol (Inderal). He reports that 5 days ago he had a temperature of 38.9º C (102º F) along with a gradual onset of diffuse abdominal pain. On examination his temperature is 38.3º C (101º F), blood pressure 100/50 mm Hg, and pulse 110/min and regular. Lungs are clear. There are numerous spider angiomata on the thorax and back, and the abdomen appears distended. Laboratory studies show: Leukocytes 13,200/mm3 (normal 5,000-10,000) Hematocrit 33% (normal 41-50%) Prothrombin time 15.2 seconds (normal 11-13 seconds) Albumin 0.1 g/dL (normal 3.4-5.4 g/dL) Sodium 135 mEq/L (normal 135-145 mEq/L) Potassium 4.7 mEq/L (normal 3.5-5.3 mEq/L) Which of the following is the most appropriate next step in diagnosis? A. Abdominal CT scan B. Abdominal paracentesis C. Abdominal ultrasound D. Electrocardiogram E. Lumbar puncture

B. Abdominal paracentesis The prevalence of infection of ascites fluid, so called spontaneous bacterial peritonitis (SBP), is estimated at 50% in hospitalized cirrhotic patients. This patient has had long-standing cirrhosis, has an extremely low serum albumin level, and is on spironolactone (a direct aldosterone antagonist), which is prescribed to patients with ascites. All febrile patients admitted with ascites must have abdominal paracentesis performed to both determine the cause for the ascites and rule out infection of the ascites. A diagnosis of SBP is made when there is an elevated ascitic fluid absolute neutrophil count (>250 cells/mm3) without an evident intra-abdominal or surgically treatable cause for the infection. SBP most often occurs with portal hypertensive ascites. The most sensitive marker available for such ascites is a serum/ascites albumin gradient >1.1 g/dL.

A patient is seen with acute abdominal pain. Abdominal film is performed, and the patient is noted to have bowel distention, air-fluid levels, and thumbprinting of the bowel wall, which reflects submucosal edema. Which of the following is the most likely condition causing the above x-ray findings? A. Acute diverticulitis B. Acute mesenteric ischemia C. Chronic mesenteric ischemia D. Acute appendicitis E. Acute diverticulosis

B. Acute mesenteric ischemia Acute mesenteric ischemia occurs from impaired blood supply to the small bowel. As a result, the small bowel becomes obstructed and has fluid coming into the bowel lumen resulting in air-fluid levels. Additionally, the small bowel wall will become edematous, with the classic sign of thumbprinting being noted on abdominal film.

You are seeing a patient admitted to the hospital with a suspected gastrointestinal (GI) problem. The patient's physical examination reveals that she has severe abdominal pain, and she is writhing in pain. Examination of the abdomen, however, is well tolerated and the patient does not have an exacerbation of the pain during this examination. Which of the following diagnoses is most likely? A. Angiodysplasia of the colon B. Acute mesenteric ischemia C. Chronic mesenteric ischemia D. Acute diverticulitis E. Acute colonic perforation

B. Acute mesenteric ischemia Patients with acute mesenteric ischemia classically have pain out of proportion to physical findings. What this means is that patients may be writhing in pain due to the ischemia, but, when the abdomen is palpated, the pain is not increased.

Certain abdominal conditions have classic descriptions associated with them. Patients who have pain out of proportion to abdominal palpation along with nausea, vomiting, distention, gastrointestinal bleeding, and altered bowel habits are likely to have which of the following diagnoses? A. Acute diverticulitis B. Acute mesenteric ischemia C. Chronic mesenteric ischemia D. Acute appendicitis E. Acute diverticulosis

B. Acute mesenteric ischemia Patients with acute mesenteric ischemia has pain out of proportion to abdominal palpation. These patients are classically described as writhing in pain but when the patient's abdomen is palpated, the patient does not have exquisite tenderness. (Although patients may be writhing in pain due to the ischemia, when the abdomen is palpated, the pain is not increased.) These patients have extreme pain because the abdominal structures are not provided with needed blood supply.

Which of the following is the most likely cell type for stomach carcinoma? A. Transitional cell B. Adenocarcinoma C. Squamous cell D. Epithelial cell

B. Adenocarcinoma Adenocarcinoma is the cell type for the development of gastric carcinoma, which usually is focal and commonly occurs in the antrum or lesser curvature of the stomach.

For patients who are diagnosed with gastric carcinoma, which of the following cell types is most likely? A. Transitional cell B. Adenocarcinoma C. Squamous cell D. Large cell E. Lymphoma

B. Adenocarcinoma Adenocarcinoma is the most common cell type of gastric cancer. Adenocarcinoma is also the most common type of colon cancer.

Which of the following tumor markers for hepatocellular carcinoma is used for both screening and monitoring response to treatment? A. Beta human chorionic gonadotropin (HCG) B. Alpha fetoprotein (AFP) C. CA-125 D. Carcinoembryonic antigen (CEA) E. CA 19-9

B. Alpha fetoprotein (AFP) AFP is the tumor maker used to assess for recurrence of hepatocellular cancer.

A male homosexual is seen in the office with complaints of fever, right upper quadrant pain, nausea/vomiting, hepatomegaly, and diarrhea. Stool antigen test for Entamoeba histolytica is positive. What is the most likely diagnosis? A. Pyogenic liver abscess B. Amebic liver abscess C. Hydatid liver cyst D. Polycystic liver cyst E. Budd-Chiari Syndrome

B. Amebic liver abscess Amebic liver abscess occurs as a result of infection by E. histolytica.

A 40-year-old woman presents with intense pruritus and fatigue. Her medical history is significant for hypothyroidism. Physical examination reveals a slightly enlarged liver and xanthomas. Laboratory studies show a total cholesterol level of 538 mg/dL and alkaline phosphatase 571 IU/L. Which of the following serum values would most likely be elevated? A. Alpha-fetoprotein B. Antimitochondrial antibody C. HBs antigen D. Prothrombin time E. Serum transaminases

B. Antimitochondrial antibody Primary biliary cirrhosis is an autoimmune disease associated with elevated antimitochondrial antibodies. Hashimoto thyroiditis, an autoimmune disease that is the most common cause of hypothyroidism, is also associated with increased levels of this antibody.

A 56-year-old alcoholic man gives a history of several years of constant epigastric pain, radiating straight through to the back. The pain is severe, present at all times, and exacerbated by eating. He also has steatorrhea and diabetes. He relates that he began to have episodes of acute alcoholic pancreatitis in his mid-thirties. At first these occurred every few years, but eventually they grew in number to several attacks per year. Eventually the pain became constant. Although he made numerous attempts to quit drinking, he was not successful until about 1 year ago; still, his current abstinence has not alleviated the pain. Which of the following is the most appropriate initial step in evaluation? A. Arteriogram B. CT scan of the upper abdomen C. Endoscopic retrograde cholangiopancreatogram (ERCP) D. Sonogram of the upper abdomen E. Upper gastrointestinal series with barium

B. CT scan of the upper abdomen The clinical diagnosis is chronic pancreatitis, and CT scan will give useful information about potentially correctable features of the disease, such as pseudocysts, dilated ducts, calculi, or areas amenable to resection. CT scan of the abdomen is considered to be the most sensitive test for picking up chronic pancreatitis. If surgery is contemplated, ERCP will then be needed.

A patient is seen in the office with complaints related to severe diarrhea. The patient has a high fever in addition to the diarrhea. The patient has just returned from a trip to the Bahamas. Which of the following is indicated at this time? A. Keeping the patient NPO until the diarrhea stops B. Ciprofloxacin (Cipro) C. Pepto-Bismol D. Amoxicillin E. Clarithromycin (Biaxin)

B. Ciprofloxacin (Cipro) When diarrhea is acquired in a resource-poor environment, this traveler's diarrhea should be treated with ciprofloxacin.

Six days after an uncomplicated left nephrectomy, a 42-year-old man complains of moderate abdominal pain and diarrhea. He is stooling approximately 6 times a day with profuse watery diarrhea. The patient also experiences painful cramps during these episodes. In other medical history, the patient has a history of hypertension and elevated cholesterol. His medications include metoprolol (Lopressor) and simvastatin (Zocor). He denies any recent cardiopulmonary history and any history of chronic gastrointestinal disorders. His chart indicates that he received a 2-day regimen of cefazolin (Ancef) for perioperative skin prophylaxis. His temperature is 38.3ºC (101.0ºF). Stool is positive for fecal leukocytes and occult blood. The diagnosis is established with additional studies and he is given treatment. Unfortunately he fails to respond to therapy and requires surgery. An image of a segment of his colon is shown. Which of the following is the most likely pathogen? A. Bacillus cereus B. Clostridium difficile C. Escherichia coli 0157:H7 D. Shigella E. Yersinia enterocolitica

B. Clostridium difficile This patient is experiencing C. difficile colitis, also known as pseudomembranous colitis. This type of diarrhea is common in hospitalized patients; at least 30% of hospitalized patients are colonized with the organism. Certain antibiotics have a predilection for causing C. difficile colitis: clindamycin, ampicillin, amoxicillin, and cephalosporins have a high incidence of causing the diarrhea related to this organism. These antibiotics suppress normal gut flora, allowing Clostridium to proliferate. Two endotoxins (A and B) cause the symptoms and the colonic pathology noted in the vignette. Diagnosis is made by ELISA test for endotoxin A and B; sigmoidoscopy also provides additional diagnostic information, particularly the presence of plaques (also called pseudomembranes). Treatment is oral metronidazole or vancomycin. Surgery is sometimes indicated for recurrences that are unresponsive to therapy or when complications such as toxic megacolon occur.

A 77-year-old woman presents to the emergency department with 12 hours of passing bright red blood from her rectum in increasingly large amounts. She becomes dizzy on standing and appears pale. Her blood pressure is 112/60 mm Hg on laying supine and 90/56 mm Hg on sitting upright with legs dangling over the side. Abdominal examination is normal. Which of the following is the most likely diagnosis? A. Bleeding diverticulum in the appendix B. Communication between an arteriole and venule in the cecum C. Laceration at the gastroesophageal junction D. Rectal polyp E. Perforation of a sigmoid diverticulum

B. Communication between an arteriole and venule in the cecum A communication between an arteriole and venule in the cecum is a description of a vascular ectasia, also known as an arteriovenous (AV) malformation. This is a common cause of painless colonic bleeding in the elderly and may present with acute gastrointestinal bleeding (as in this case), chronic gastrointestinal bleeding, or iron-deficiency anemia. These lesions may be difficult to demonstrate, as the bleeding may be intermittent or the colon may be so full of blood that the site of origin is obscured. Techniques used to demonstrate bleeding AV malformations include colonoscopy, intraoperative endoscopy, and visceral angiography. Treatment of these lesions is problematic, because many patients will subsequently develop new or recurrent bleeding vessels.

On postoperative day 5 after elective left hemicolectomy for chronic diverticular disease, a patient is draining copious amounts of clear pink fluid from his midline laparotomy wound. The dressing is removed revealing that it is soaked along with a normal-appearing fresh wound with a row of skin staples in place. When attempting to get out of bed, the patient's wound opens widely, and a handful of small bowel suddenly rushes out.Which of the following is the most appropriate management at this time? A. Cover bowel with dry sterile dressings and schedule urgent surgical closure B. Cover bowel with sterile dressings soaked in warm saline and rush patient to the operating room C. Irrigate bowel with cold antiseptic solutions while awaiting urgent surgical closure D. Take patient to the treatment room and suture the skin edges together E. Wearing sterile gloves, push bowel back in and tape the wound securely

B. Cover bowel with sterile dressings soaked in warm saline and rush patient to the operating room Until the patient attempted to get out of bed, he had a wound dehiscence that could have been handled by taping the wound securely. Once the bowel came out, the problem became an evisceration. Immediate surgical repair is mandatory. While setting it up, the bowel must be protected from drying out, and the patient must be protected from significant heat loss. The key for immediate treatment is application of warm and moist dressings.

A patient with long-standing Crohn disease has recurrent flares that do not respond to medical management. The patient ultimately has an ileal resection, which results in bile salt malabsorption. Which of the following clinical manifestations is consistent with this bile salt deficiency state? A. Flushing due to serotonin loss B. Diarrhea C. Heartburn D. Melena E. Fecal incontinence

B. Diarrhea Bile salt deficiency from ileal resection results in diarrhea due to the loss of surface area.

Patients with ascites and advanced liver disease may have hepatic encephalopathy as a complication of this condition. All of the following are treatments or prevention strategies for hepatic encephalopathy EXCEPT: A. Lactulose B. Diuretics C. Low-protein diets D. Neomycin

B. Diuretics Diuretics may cause hepatic encephalopathy because they can induce alkalosis, which is a known precipitant.

A patient with a history of chronic intake of alcohol and smoking cigarettes is seen in the office. The patient would like to have an esophagogastroduodenoscopy done because his uncle was just diagnosed with esophageal cancer. Which of the following signs or symptoms is considered to be the earliest complaint/finding in a patient with esophageal cancer? A. Dysphagia for liquids B. Dysphagia for solids C. Jaundice D. Early satiety E. Odynophagia

B. Dysphagia for solids Dysphagia for solids and weight loss are the earliest findings with esophageal cancer.

Which of the following causes of infectious diarrhea can be complicated by hemolytic-uremic syndrome (HUS) and thrombotic thrombocytopenia purpura (TTP)? A. Enterotoxigenic E. coli B. E. coli 0157:H7 C. Vibrio cholerae D. Staphylococcal food poisoning E. Salmonella

B. E. coli 0157:H7 E. coli 0157:H7 can result in HUS and TTP. This Shiga-toxin producing strain of E. coli is the most common cause of HUS and kidney-related disease in children under age 3.

Infection with Helicobacter pylori is associated with all of the following clinical entities EXCEPT: A. Chronic gastritis B. Esophageal cancer C. Atrophic gastritis D. Gastric cancer E. Peptic ulcer disease

B. Esophageal cancer H. pylori colonizes gastric antral mucosa, produces urease, and causes active gastritis and atrophic gastritis. This environment lessens gastric acid output, which lowers the risk for esophageal cancer since refluxed material will be less acidic and less likely to lead to Barrett esophagus with changes from squamous to columnar cells.

A 4-year-old girl was diagnosed with left-sided otitis media 10 days ago and prescribed oral amoxicillin 40 mg/kg/day for 7 days. She has since developed watery diarrhea with mucus, crampy abdominal pain, and fever. On physical examination her temperature is 39.4°C (102.9°F), pulse 88/min, and respirations 16/min. She has normal bowel sounds and is diffusely tender to palpation. Which of the following is the most appropriate initial step in diagnosis? A. Barium enema B. Evaluation of stool for Clostridium difficile toxins C. Evaluation of stool for rotavirus D. Stool examination for ova and parasites E. Stool Hemoccult test

B. Evaluation of stool for Clostridium difficile toxins The child has pseudomembranous colitis, caused by the toxins produced by Clostridium difficile. The colon is colonized by C. difficile after eradication of the normal microflora by a course of antibiotics. Virtually all antibiotics have been implicated in the pathogenesis of pseudomembranous colitis. Patients usually present with watery diarrhea, tenesmus, fever, and abdominal pain. Symptoms typically occur 7-10 days after initiation of the antibiotic; in rare cases, pseudomembranous colitis can occur up to 6 weeks after antibiotic initiation. Toxic megacolon, resulting from progressive loss of muscular tone in the colon, is a rare but serious complication that could progress to perforation of the colon and peritonitis. On sigmoidoscopy or colonoscopy, the disease is characterized by an accumulation of an inflammatory exudate, the "pseudomembrane" over the mucosa. Assay for C. difficile toxins (enterotoxin A or cytotoxin B) is useful in diagnosing pseudomembranous colitis. Oral metronidazole is the treatment of choice but another option is oral vancomycin. Barium enema (choice A) can image gross structural abnormalities of the colon but is not helpful in diagnosing pseudomembranous colitis.

A 9-month-old boy is brought to the health care provider's office because his weight is persistently below the tenth percentile. His mother states that the infant seems to be hungry all the time and usually consumes 8-12 oz of formula every 2-3 hours in addition to some table food. He also has frequent, bulky, and malodorous stools. He does not take any medications. A malabsorption syndrome is suspected. The results of which of the following tests will most likely be abnormal? A. Abdominal radiography B. Fecal fat quantification C. Serum albumin D. Stool culture for Clostridium difficile toxins E. Stool smear for leukocytes and eosinophils

B. Fecal fat quantification The most useful test for the 9-month-old infant in this clinical vignette is fecal fat quantification. Fat malabsorption is likely, with poor weight gain and frequent malodorous stools. A detailed family history and physical examination are also useful in evaluation of fat malabsorption syndrome. In a stool sample collected over 3-4 days, fat excretion should not exceed 15% of dietary fat in an infant, or 10% in an older child. When a malabsorption syndrome is confirmed, a sweat chloride test is indicated to evaluate for the possibility of cystic fibrosis, which is a common cause of fat malabsorption.

A patient undergoes a colonoscopy, revealing multiple polyps. This patient also has multiple sebaceous cysts, multiple benign soft tissue tumors, and desmoid tumors. Which of the following is the most likely diagnosis? A. Familial adenomatous polyposis B. Gardner syndrome C. Turcot syndrome D. Peutz-Jeghers syndrome E. Hereditary nonpolyposis colorectal cancer

B. Gardner syndrome Gardner syndrome has multiple colonic polyps, sebaceous cysts, multiple benign soft tissue tumors, dental abnormalities, and desmoid tumors. This condition is one of the familial adenomatous polyposis conditions that encompasses these additional extracolonic manifestations.

Which of the following hereditable gastrointestinal (GI) polyposis syndromes has associated osteomas, fibromas, lipomas, and epidermal cysts? A. Familial adenomatous polyposis B. Gardner syndrome C. Lynch syndrome (nonpolyposis syndrome) D. Peutz-Jeghers syndrome E. Juvenile polyposis

B. Gardner syndrome Gardner syndrome is a variant of familial polyposis coli that is associated with soft tissue tumors including epidermoid cysts, osteomas, lipomas, fibromas, and desmoid tumors.

Which of the following types of cancer is most commonly associated with a palpable Blumer shelf, left supraclavicular or scalene adenopathy, and periumbilical mass? A. Esophageal cancer B. Gastric cancer C. Colorectal cancer D. Liver cancer E. Cholangiocarcinoma

B. Gastric cancer Gastric cancer metastatic spread can be manifested by hepatomegaly; ascites; left supraclavicular or scalene adenopathy; periumbilical, ovarian, or palpable prerectal mass (Blumer shelf); and skin manifestations, such as nodules and multiple seborrheic keratoses.

Which of the following is the most common sign or symptom seen with rectal cancer? A. Melena B. Hematochezia C. Distended bowel from obstruction D. Fat-laden stools which float on the top of the water in the toilet bowl E. Weakness due to anemia

B. Hematochezia Hematochezia is the most common symptom of rectal cancer. Hematochezia is blood covering the stool as it passes through the colon. Hematochezia is seen as a result of the colon cancer in the rectal area bleeding.

A 74-year-old man comes to the emergency department after two episodes of passing large amounts of bright red blood per rectum that occurred without pain. He has a long history of chronic constipation without a recent change in his bowel habits. A flexible sigmoidoscopy reveals multiple large diverticula in the sigmoid colon with copious amounts of fresh blood in this region. No other abnormalities are seen on examination of the splenic flexure, where formed brown stool is encountered. Which of the following is the most likely underlying cause for his bleeding? A. Diffuse descending colon ischemia B. Hemorrhage from a single diverticular arteriole C. Oozing from an inferior mesenteric vein D. Thrombosis of a branch of the inferior mesenteric artery E. Thrombosis of internal hemorrhoids

B. Hemorrhage from a single diverticular arteriole One of the important complications of diverticular disease is massive bleeding that can be potentially life-threatening. The mechanism involved in producing bleeding in diverticular disease is usually the disruption of an often single arteriole or small artery in a diverticulum. The diverticulum involved is often, but not always, particularly large. The small vessel can be sufficiently stretched by the diverticulum that it cannot undergo contraction, which normally stops blood flow through this size vessel after damage. In many cases, the bleeding may stop spontaneously. If it recurs within a few days, surgical resection of the involved segment of bowel (if it can be identified, which can be problematic) may be required.

A patient is admitted to the hospital with suspected appendicitis. You are performing the medical clearance for surgery because the patient has long-standing hypertension and diabetes. As part of your patient education, you tell the patient that the most common cause of acute appendicitis in the United States is which of the following? A. Fecalith B. Hyperplasia of lymphoid tissue C. Parasite D. Carcinoid tumor E. Klatskin tumor

B. Hyperplasia of lymphoid tissue Hyperplasia of lymphoid tissue accounts for 60% of acute appendicitis cases, as it leads to obstruction of the lumen of the appendix.

A patient is admitted to the hospital with tachypnea, tachycardia, and orthostasis. In a patient with previously normal renal function, which of the following laboratory results will be consistent with an upper gastrointestinal (GI) bleed? A. Increased serum creatinine B. Increased blood urea nitrogen (BUN) to creatinine ratio C. Hyponatremia D. Hypercalcemia E. Hyperamylasemia

B. Increased blood urea nitrogen (BUN) to creatinine ratio Acute upper GI bleed will cause an increased BUN to creatinine ratio. This occurs as a result of the protein portion of the blood increasing the BUN.

A patient is admitted to the hospital for elective cholecystectomy because she was found to have a porcelain gallbladder. For which of the following reasons is elective cholecystectomy performed? A. Increased risk of cholangitis B. Increased risk of gallbladder cancer C. Increased risk of sclerosing cholangitis D. Increased risk of pancreatitis E. Increased risk of hepatic abscess

B. Increased risk of gallbladder cancer Elective cholecystectomy is pursued in setting of porcelain gallbladder because 50% of patients go on to develop gallbladder cancer.

A 43-year-old man with recurrent history of peptic ulcer disease associated with diarrhea and strong family history of duodenal ulcer disease is suspected of having Zollinger-Ellison syndrome (gastrinoma). Secretin (1 U/kg) is given as a rapid IV injection to test for gastrinoma. Which of the following results would support the existence of gastrinoma following secretin administration? A. Gastrin release from antrum B. Increased serum gastrin C. Inhibition of gastric emptying D. Inhibition of gastric secretion E. Stimulation of pancreatic HCO3- secretion

B. Increased serum gastrin Gastrinomas are gastrin-secreting tumors usually present in the pancreas. Patients who have gastrinoma have high serum gastrin levels, which lead to hypersecretion of gastric acid and consequent duodenal and jejunal ulcers. Injection of secretin is the most specific and easiest test for gastrinoma.

Brown pigmented gallstones are associated with which of the following conditions? A. Hemolysis B. Infection C. Obesity D. Ileal resection E. Crohn disease

B. Infection Brown pigmented stones seen in bile ducts are most commonly associated with biliary tract infection.

Colic symptoms are not the same for gallbladder colic and ureteral colic. Signs and symptoms of biliary colic include all of the following EXCEPT: A. Right upper quadrant pain that occurs suddenly B. Intermittent right upper quadrant pain C. Pain lasting several hours D. Abdominal pain that may radiate into the right scapula or back E. Pain occurring 30 to 90 minutes following meal

B. Intermittent right upper quadrant pain Unlike ureteral colic, biliary colic pain is a constant pain lasting several hours rather than intermittent abdominal pain, which is what happens with ureteral colic caused by a renal stone.

A patient is seen post-procedure after having a rubber band ligation in the anal area. Which of the following conditions was treated? A. External hemorrhoids B. Internal hemorrhoids C. Anal fissure D. Anal fistula E. Anal cancer

B. Internal hemorrhoids Rubber band ligation is an office-based treatment of choice for stage 1 or stage 2 internal hemorrhoids that are bleeding. This procedure is painless, effective, and well-tolerated.

A patient is evaluated for cholestasis symptoms and found to have bead-like structuring and bead-like dilations of the intrahepatic and extrahepatic ducts. Which of the following is the curative treatment for this patient? A. Cholecystectomy B. Liver transplantation C. Resection of the large bowel D. Ursodiol (Actigall) E. Kidney transplant

B. Liver transplantation The only curative possibility for primary sclerosing cholangitis is liver transplant.

A 79-year-old man with atrial fibrillation develops an acute abdomen. When seen 2 days after the onset of the abdominal pain, he has a silent abdomen, with diffuse tenderness and mild rebound. There is a trace of blood noted in the stool that was obtained on the rectal examination. He also has acidosis and looks quite sick. Radiographs show distended small bowel and distended right colon, up to the middle of the transverse colon.Which of the following is the most likely diagnosis? A. Acute pancreatitis B. Mesenteric ischemia C. Midgut volvulus D. Perforated viscus E. Primary peritonitis

B. Mesenteric ischemia The setting of an elderly patient who has atrial fibrillation, atherosclerosis, or a recent myocardial infarction and who develops an acute abdomen strongly suggests mesenteric ischemia. This ischemia can be the result of atherosclerosis with poor blood supply to the gut or can be caused by an embolus to the mesenteric vessels. The combination of abdominal pain and a trace of blood in the lumen is also typical, as is the radiograph outlining the territory supplied by the superior mesenteric artery. The patient probably has a dead bowel by now, as evidenced by his acidosis and severe illness. Any one of the other options could exist, but none are the most likely.

Which of the following medications is typically used to treat hepatic encephalopathy? A. Cholestyramine (Questran) B. Neomycin C. Furosemide (Lasix) D. Activated protein C E. Pentoxifylline (Trental)

B. Neomycin Neomycin and metronidazole are poorly absorbed antibiotics that decrease the number of gut bacteria and therefore lower protein formation, which helps with hepatic encephalopathy. Rifaximin may also be used in this setting as another way of lowering the amount of gut bacteria so that less protein is produced.

A patient presents with gastrointestinal complaints of chronic episodic diarrhea. The patient also complains of intermittent wheezing and flushing with the diarrhea. Which of the following treatment modalities is used in order to treat these symptoms? A. Nonsteroidal anti-inflammatory drugs (NSAIDs) B. Octreotide C. Beta-blockers D. Calcium channel blockers E. Angiotensin-converting enzyme (ACE) inhibitors

B. Octreotide Carcinoid syndrome is treated with surgical resection when possible, while symptoms are controlled with octreotide, histamine blockers, and selective serotonin-reuptake inhibitors (SSRIs). Metastatic carcinoid syndrome is treated with hepatic artery embolization and chemotherapy.

Advanced liver disease will result in prolongation of which of the following? A. Activated partial thromboplastin time (aPTT) B. Prothrombin time C. Thrombin time D. Bleeding time E. Platelet count

B. Prothrombin time The liver makes the clotting factors, which primarily affect the extrinsic clotting system. When the clotting factors of the extrinsic system are not made in the setting of end-stage liver disease, the prothrombin time is prolonged.

Which of the following treatments is first-line therapy for a patient with gastric complaints due to Zollinger-Ellison (Z-E) syndrome? A. Histamine-2 blockers B. Proton-pump inhibitors (PPIs) C. Sucralfate D. Misoprostol E. Sodium bicarbonate

B. Proton-pump inhibitors (PPIs) High-dose proton-pump inhibitors (PPIs) are the treatment of choice for Z-E syndrome in order to lower maximum gastric ulcer output.

A patient is admitted due to lower gastrointestinal bleeding caused by angiodysplasia of the colon (arteriovenous [AV] malformations). If the patient does not respond to colonoscopic coagulation of the lesion, which of the following clinical interventions is indicated? A. Coiling of the involved vessel B. Right hemicolectomy C. Left hemicolectomy D. Diverting colostomy E. Abdominal perineal resection

B. Right hemicolectomy Right hemicolectomy is done for patients with AV malformation bleeding not responding to colonoscopic coagulation of the lesion. Surgery is the definitive therapy for angiodysplasia that does not respond to endoscopic treatment.

A patient presents with gastrointestinal complaints of chronic episodic diarrhea. The patient also complains of intermittent wheezing and flushing with the diarrhea. Laboratory evaluation for this patient would specifically include which of the following? A. Stool for fecal fat B. Serum 5-HIAA level C. Alpha-fetoprotein level D. Serum amylase level E. Tissue transglutamate level

B. Serum 5-HIAA level Patients with carcinoid tumors have increased production of the serotonin metabolite 5-HIAA, which is detected in the urine.

Which of the following is a known complication of long-standing achalasia? A. Adenocarcinoma of the esophagus B. Squamous cell carcinoma of the esophagus C. Plummer-Vinson syndrome D. Esophageal rings E. Schatzki ring

B. Squamous cell carcinoma of the esophagus Squamous cell carcinoma of the esophagus is a complication of achalasia.

Which of the following pathogens identified in the bloodstream has been linked with the development of colorectal cancer? A. Streptococcus pneumoniae B. Streptococcus bovis C. Staphylococcus epidermidis D. Mycoplasma E. Mycobacterium tuberculosis

B. Streptococcus bovis S. bovis bacteremia is associated with colon malignancy, especially right-colon cancer.

266. Cholangiocarcinoma typically occurs as a complication of which of the following conditions? A. Crohn disease B. Ulcerative colitis C. Diverticulitis D. Hepatitis E. Cirrhosis

B. Ulcerative colitis The correct answer is B. Cholangiocarcinoma is a complication of sclerosing cholangitis, which is primarily seen in the setting of ulcerative colitis.

A patient is admitted to the hospital with tachycardia, dyspnea, and orthostasis. The patient complained of melena, and a rectal examination confirmed blood in the stool. After ensuring that the patient is stable from ongoing resuscitation, which of the following is the next step in his evaluation? A. Plain film of the abdomen B. Upper endoscopy C. Push procedure D. Colonoscopy E. Anoscopy

B. Upper endoscopy Upper endoscopy is the initial test done for patients with melena. Melena typically is gastrointestinal (GI) bleeding occurring in the upper GI tract.

Patients with chronic pancreatitis are at increased risk for the development of deficiency for which of the following? A. Glucose B. Vitamin B12 C. Vitamin B1 (thiamine) D. Vitamin B6 E. Vitamin C

B. Vitamin B12 Vitamin B12 is low in the setting of chronic malabsorption with chronic pancreatitis. Patients with chronic pancreatitis and an exocrine pancreatic deficiency state will have a reduction in intestinal pH, which interferes with vitamin B12 binding with intrinsic factor, leading to vitamin B12 deficiency.

Patients with familial polyposis coli may be given which of the following medications in order to cause regression of polyps and also inhibit their development? A. Estrogen B. Calcium C. Nonsteroidal anti-inflammatory drugs (NSAIDs) D. Proton-pump inhibitors (PPIs) E. Allopurinol

C. Nonsteroidal anti-inflammatory drugs (NSAIDs) NSAIDs cause regression of polyps and also inhibit their development. This treatment may be pursued prior to prophylactic total colectomy, which is the definitive therapy for familial polyposis coli, as patients may have 100% chance of developing colon cancer because of the multiple polyps.

Four weeks after a camping trip, a 16-year-old boy begins to pass foul-smelling stools. He also develops anorexia and flatulence. None of his friends from the trip are ill. He is generally healthy and takes no medications. His temperature is 37.3ºC (99.1ºF). Physical examination shows diffuse abdominal pain and distention and guaiac-negative stool. The remainder of the examination is unremarkable. A microscopic image of his stool is shown. Which of the following statements is most accurate regarding his condition?? A. Asymptomatic infection is rare B. Boiling of water during the camping trip would not have killed the infective organism C. A 5- to 7-day course of metronidazole (Flagyl) has a cure rate of 80 to 95% D. Infection is limited to the large intestine E. The trophozoite is the infective form of this organism

C. A 5- to 7-day course of metronidazole (Flagyl) has a cure rate of 80 to 95% This patient has giardiasis, which is caused by Giardia lamblia. G. lamblia is a flagellated protozoan that exists in a cyst and trophozoite form. The cyst, not the trophozoite, is the infective form (choice E). Infection is limited to the biliary tract and small intestine, not the large intestine (choice D). Contaminated water supplies are the common source of infection, and boiling water can kill the infective cysts (choice B). Many individuals infected with Giardia remain asymptomatic (choice A). A 5- to 7-day course of metronidazole has a cure rate of 80 to 95%.

90. A patient tests positive for Clostridium difficile colitis. The patient is started on appropriate treatment as part of the outpatient therapy but continues to have diarrhea. Which of the following treatments should be pursued A. Add high-fiber diet B. Add probiotics C. Add cholestyramine (Questran) D. Add fruit-based diet E. Perform fecal transplant

C. Add cholestyramine (Questran) The correct answer is C. As adjunctive therapy to the appropriate antibiotic given for C. difficile colitis, cholestyramine is added to the treatment regimen in order to improve diarrhea. This agent acts as a bile-acid sequestrant and can help to form solid stool.

A patient with long-standing Crohn disease presents to the office with right-sided flank pain and hematuria. Which of the following is most likely responsible for this presentation? A. Cysteine stones B. Struvite stones C. Calcium oxalate stones D. Uric acid stones E. Acute pyelonephritis

C. Calcium oxalate stones Calcium oxalate stones are seen in patients with Crohn disease because of increased dietary absorption of oxalate and altered enterohepatic circulation.

Which of the following causes of diarrhea is considered to be the most common cause of bacterial diarrhea in the United States? A. Enterotoxigenic E. coli B. E. coli 0157:H7 C. Campylobacter jejuni D. Staphylococcal food poisoning E. Salmonella

C. Campylobacter jejuni Campylobacter is the most common cause of bacterial diarrhea in the United States. This condition can be complicated by myasthenia gravis. It is typically acquired by the ingestion of contaminated, undercooked poultry and can cause both bloody and watery diarrhea.

A 56-year-old man presents with complaints of gnawing pain in the mid-epigastrium, with occasional radiation to the back. He also notes a 15-pound weight loss over the past 3 months. The clinician suspects pancreatic carcinoma. Which of the following tumor markers would most likely to be seen in this condition? A. Alpha-fetoprotein (AFP) B. CA-125 C. Cancer Antigen (CA 19-9) D. Human chorionic gonadotropin (hCG) E. Prostate-specific antigen (PSA)

C. Cancer Antigen (CA 19-9) Tumor markers can be very helpful in narrowing the possible primary sources for metastatic lesions. CA 19-9 is present in about 80% of patients who have pancreatic cancer. Tumor markers should not be used as the primary tool for cancer screening, but they have considerable utility in the confirmation of the diagnosis, as well as for monitoring recurrence or response to therapy.

A 56-year-old man has been having bloody bowel movements on and off for the past several weeks. He reports that the blood is bright red, coats the outside of the stools, and appears in the toilet bowl even before he wipes himself. There is also blood on the toilet paper after he wipes. After further questioning it is ascertained that he has been constipated for the past 2 months and that the caliber of the stools has changed. They are now very thin compared to the usual diameter of an inch or so that was customary for him. He has some minor discomfort. Which of the following is the most likely diagnosis? A. Anal fissure B. Cancer of the cecum C. Cancer of the rectum D. External hemorrhoids E. Internal hemorrhoids

C. Cancer of the rectum The combination of red blood coating the stools and a change in bowel habit and stool caliber spells out cancer of the rectum in someone in this age group. The stools become smaller in diameter as a result of the rectal tumor encroaching on the bowel lumen causing the stools to become smaller.

For symptomatic patients who have colorectal cancer, which of the following is the earliest manifestation? A. Easy bruising B. Increased flatulence C. Change in bowel habits D. Jaundice E. Tenderness of the liver

C. Change in bowel habits For patients who develop symptoms of colorectal cancer, the most common symptom that occurs is a change in bowel habits. Since this symptom is nonspecific, colorectal cancer that is found only after the development of symptoms is more likely to be advanced. The change in bowel habits is seen more commonly in left-sided colon cancers, whereas occult anemia is a more common presentation for patients with right-sided colon cancer.

A patient is seen in the outpatient clinic. The patient is 60 years old and completed the fecal occult blood card testing and one of three cards was positive for occult blood. Which of the following is the next step in his evaluation? A. Flexible sigmoidoscopy B. Barium enema C. Colonoscopy D. CT of abdomen E. Carcinoembryonic antigen (CEA) test

C. Colonoscopy Colonoscopy is the most sensitive and specific test and is indicated for any positive fecal occult blood test.

Which of the following is the biggest risk for the development of hepatocellular adenomas? A. Chronic ingestion of significantly hot tea B. Chronic hepatitis C infection C. Oral contraceptives D. Fulminate hepatitis A infection E. Non-alcoholic steatohepatitis (NASH)

C. Oral contraceptives Hepatocellular adenomas occur most commonly in women in their third or fourth decade who take birth control pills.

A 69-year-old woman comes to the health care provider for a periodic health maintenance examination. She has no complaints. She takes 1,500 mg of calcium daily, a multivitamin, and daily aspirin. She exercises regularly, eats a fairly healthy diet, and does not smoke cigarettes. She drinks a couple of glasses of wine with dinner each evening and smokes marijuana occasionally with a friend who has glaucoma. She is concerned about her risks for breast, ovarian, and colon cancer because she has friends with each of these diseases. Breast and pelvic examination are unremarkable. A mammogram is normal. Findings from a barium enema are shown. Which of the following is the most common presenting symptom of this condition? A. Anemia B. Blumer shelf C. Constipation D. Virchow node E. Weight loss

C. Constipation The barium enema demonstrates an irregular lesion that causes obstruction in the sigmoid colon. This finding on barium enema is highly suspicious for adenocarcinoma. Cancers arising in the left and sigmoid colon are characterized by rectal bleeding, obstructive symptoms, and constipation possibly alternating with diarrhea. Stools may develop a narrow caliber as the tumor encroaches onto the lumen.

A 44-year-old obese woman comes to the emergency department complaining of 3 hours of severe abdominal pain. She has also had multiple episodes of vomiting during this time. She describes the pain as "worse than labor," and it radiates to the interscapular region. Her temperature is 38.9ºC (102ºF), and there is severe tenderness in the right upper quadrant. She reports that she has had multiple similar episodes in the past which have lasted approximately 30 minutes and then resolved spontaneously. Which of the following locations is most likely being obstructed by a gallstone? A. Common bile duct B. Common hepatic duct C. Cystic duct D. Pancreatic duct E. Right hepatic duct

C. Cystic duct This patient has acute cholecystitis. Risk factors include female gender, obesity, and a classic history of prolonged biliary colic in association with fevers. The presentation illustrated here is typical and results from obstruction of the cystic duct, which drains the gallbladder.

A 44-year-old woman comes to the clinic after being treated in the emergency department 2 weeks ago for her first episode of diverticulitis. Her condition was diagnosed by CT scan of the abdomen and treated with an outpatient course of ciprofloxacin (Cipro) and metronidazole (Flagyl). She has no previous medical history and takes no medication on a regular basis. The patient's recovery was uneventful, and she is currently without any new complaints. She is now concerned about multiple small gallstones that were incidentally noted on the CT scan. Of note, the gallbladder appeared otherwise normal, and there was an absence of dilated ducts. She denies any problems with cholelithiasis in the past but is worried because her mother had a "gallstone infection" in the past that required surgery and intravenous antibiotics. Her temperature is 37.0ºC (98.6ºF), blood pressure 140/80 mm Hg, pulse 93/min, and respirations 18/min. She is 60 inches tall and weighs 77 kg (169 lb). Her abdomen is nontender and nondistended with normal bowel sounds. There is no rebound or guarding. Murphy sign is negative. Her liver is normal sized. Laboratory studies show normal levels for all of the following: Hemoglobin 11 mg/dL Leukocyte count 5700/mm3 Platelets 270,000/mm3 Hematocrit 31% Sodium 145 mEq/L Potassium 4.5 mEq/L Chloride 100 mEq/L Bicarbonate 24 mEq/L Glucose 116 mg/dL Creatinine 1.0 mg/dL BUN 10 mg/dL Total protein 6.0 g/dL Albuimin 3.5 g/dL Total bilirubin 0.9 mg/dL Direct bilirubin 0.4 mg/dL Alkaline phosphatase 59 U/L AST 20 U/L ALT 31 U/L Which of the following is the most appropriate management of this patient? A. Elective laparoscopic cholecystectomy B. Endoscopic sphincterotomy C. Exercise and diet modification D. Right upper quadrant ultrasound E. Treatment with ursodeoxycholic acid

C. Exercise and diet modification This patient has asymptomatic gallstones that were incidentally found. Gallstones are frequently asymptomatic and when found do not require any further intervention or workup. There is some evidence that a low-carbohydrate diet together with increased physical activity may help prevent gallstone formation. In our patient, lifestyle modification would be useful mainly to prevent the sequela of obesity. Rapid weight loss should be discouraged in patients who have gallstones, because it may actually increase the risk for symptomatic gallstone formation caused by the biliary sludge that might occur in the setting of a patient eating fewer fat calories.

Signs, symptoms, and laboratory findings consistent with cholestasis include all of the following EXCEPT: A. Pruritus B. Jaundice C. Grossly elevated aspartate transaminase (AST)/alanine transaminase (ALT) D. Grossly elevated alkaline phosphatasezElevated serum cholesterol E. Elevated serum cholesterol

C. Grossly elevated aspartate transaminase (AST)/alanine transaminase (ALT) Cholestasis increases alkaline phosphatase levels due to gallbladder obstruction or sludge in the gallbladder, but the transaminase levels are not elevated. Elevated transaminase levels are most commonly seen in the setting of acute infection such as hepatitis.

A 44-year-old school bus driver comes to the emergency department complaining of severe abdominal pain. She reports that the pain began approximately 8 hours ago, after eating lunch at a fast-food restaurant. The pain has become increasingly severe and radiates to her back. She recalls a similar episode, lasting 3 hours, 2 months earlier and another episode, lasting 12 hours, during her last pregnancy. She is afebrile, and without jaundice, and has right upper quadrant tenderness with deep palpation of this area. A rectal examination reveals brown stool that is negative for occult blood. Her leukocyte count is 12,900/mm3, (normal 5,000-10,000) and her hematocrit is 39% (normal 41-50%). Her total bilirubin is 2.1 mg/dL (normal <1.9). An ultrasound is performed at the bedside revealing a positive ultrasound Murphy sign but there were no stones or sludge noted in the gallbladder and the cystic duct is not dilated. Which of the following is the most appropriate next step in her diagnostic evaluation? A. CT scan of the abdomen and pelvis B. Endoscopic retrograde cholangiopancreatography (ERCP) C. HIDA scan D. Percutaneous transhepatic cholangiogram (PTC) E. Upper gastrointestinal barium study

C. HIDA scan This patient has the classic presentation of acute cholecystitis. The episodes she had several months ago and during pregnancy suggest a prior history of biliary colic. A HIDA scan is a noninvasive nuclear medicine test that will reveal obstruction of the cystic duct, which is caused by an impacted gallstone in the cystic duct, and is the cause of acute cholecystitis. The patient's elevated total bilirubin provides evidence that the bile duct is blocked. The HIDA scan can also be used to identify gallbladder dysfunction for patients whose gallbladders do not function but who may not have stones.

Patients with cirrhosis may go on to develop ascites. In addition to increased portal pressures, which of the following conditions provides the other explanation as to why ascites develops in this situation? A. Lack of clotting factors produced by the liver B. Overhydrating the patient C. Hypoalbuminemia D. High-salt diet E. Continued intake of alcohol

C. Hypoalbuminemia Ascites arises because of hypoalbuminemia and increased portal pressures. Without enough protein and albumin in the circulatory system, there is a leakage of fluid out of the cells, further contributing to ascites.

76. A 9-year-old boy is brought to the emergency department by his grandmother because he suddenly developed a fever with shallow breathing, vomiting, and mental confusion. She is beside herself because the child has come to visit over the holidays and had been perfectly well before today. He has never had any problems with his health except for a skin condition present from birth that she describes as "alligator skin." When you ask if she gave her grandson any unusual medication or remedy, she admits to having applied a homemade paste to his skin that was supposed to get rid of the heavy scale. She states that there were several affected family members in previous generations and this remedy has been used to treat them without ever having caused problems. She also insists that it could not have been the cause of any illness, because it was really just a bunch of aspirin tablets crushed and mixed with petrolatum. The boy's temperature is 39ºC (102.2ºF), pulse 120/min, and respirations 35/min. He has vomited twice while in the emergency department and appears to be dehydrated and lethargic. Blood glucose is 45 mg/dL and serum pH is 6.9. Which of the following is the most appropriate next step? A. Activated charcoal by mouth B. Gastric lavage C. Intravenous bicarbonate D. Intravenous 5% dextrose in saline E. Oral N-acetylcysteine

C. Intravenous bicarbonate Salicylates have analgesic and antipyretic properties when used systemically, and keratolytic properties when used topically. This patient has developed salicylate intoxication secondary to absorption of large amounts of salicylate through the skin. Many skin conditions that are characterized by hyperkeratosis are treated with topical preparations that contain a certain amount of salicylic acid (ranging from 1 to 40%, depending on the indication). They should never be used in children, however, because the surface-to-volume ratio is very large and even a relatively small amount absorbed through a large surface area will result in significant systemic levels. Even in adults, preparations with a significant amount of salicylic acid should be used cautiously, and either for a very limited time or only on certain body surface areas at a time. The clinical presentation and physical findings will vary with the dose of salicylates ingested. Vomiting, hyperpnea, fever, lethargy, and mental confusion are seen in mild salicylate poisoning. Convulsions, coma, and respiratory and cardiovascular collapse are seen in severe salicylate poisoning. Hyperventilation, dehydration, bleeding disorders, seizures, and coma are seen in chronic salicylate ingestion. Arterial blood gas analysis will show metabolic acidosis with respiratory compensation in children and a respiratory alkalosis alone in adolescents. To enhance the excretion of salicylate, the intravenous route should be used to administer bicarbonate and raise the urine pH to 7.0-7.5. If the salicylate level is greater than 100 mg/dL, hemodialysis may be indicated.

A 28-year-old man with end-stage renal disease (ESRD) on continuous ambulatory peritoneal dialysis (CAPD) for 2 months is brought to the emergency department with fever, abdominal pain, and cloudy dialysis fluid. The pain has been present for 12 hours. There is no diarrhea or vomiting. The patient has ESRD secondary to chronic glomerulonephritis; there is no history of diabetes, urinary infections, or antibiotic use. Examination reveals a temperature of 38.9ºC (102ºF) and blood pressure 110/70 mm Hg. The throat is clear, as are the lungs. Cardiac examination reveals a grade 2/6 systolic murmur. Abdominal examination reveals decreased bowel sounds with diffuse tenderness. There is mild rebound tenderness. There is no edema or skin rash. A complete blood count shows a leukocyte count of 14,200/mm3 (normal 5,000-10,000/mm3) and hemoglobin 12.5 g/dL (normal 13.8-17.0 g/dL). Peritoneal fluid is cloudy with 1,000 white blood cells, 85% of which are polymorphonuclear leukocytes. Gram stain of the peritoneal fluid is negative. Cultures of blood and peritoneal dialysis fluid are taken. Which of the following is the most appropriate initial step in management? A. Fluconazole B. Immediate removal of dialysis catheter C. Intravenous ceftriaxone and vancomycin D. Intravenous gentamicin E. Oral ciprofloxacin

C. Intravenous ceftriaxone and vancomycin Gram-negative organisms classically cause peritonitis in a patient on CAPD, but there is a recent tendency for this condition to be caused by gram-positive pathogens such as Staphylococcus aureus or Epidermidis. It is usually characterized by abdominal pain and over 100 white blood cells (typically polymorphonuclear leukocytes) in a sample of peritoneal dialysis fluid. Intravenous ceftriaxone and vancomycin would be a reasonable treatment to cover both gram-negative and gram-positive pathogens.

Patients with signs and symptoms of cholecystitis can have obstruction at multiple points in the gastrointestinal (GI) tract. Which of the following signs or symptoms in a patient with known cholelithiasis would make a health care provider suspect common duct stones? A. Inability to tolerate oral intake B. Weight loss C. Jaundice D. Persistent nausea and vomiting E. Radiation of abdominal pain into the scapula

C. Jaundice Jaundice is a clinical sign signifying that the gallstone is lodged in the common duct rather than in the cystic duct. A stone in the common duct will block the outflow of other liver and pancreatic enzymes into the gastrointestinal tract. The health care provider should suspect common bile duct stones in a patient who has jaundice, pancreatitis, abnormal liver function tests, or ultrasound revealing a dilated common bile duct or stones in the duct.

A 36-year-old woman without significant past medical history is complaining of 8 weeks of intermittent diarrhea. She reports the diarrhea is nonbloody and without mucus. There is mild abdominal cramping but no severe pain. There is no associated constipation, and symptoms are not worse during stressful periods. The diarrhea occurs shortly after meals and improves with fasting. She denies any weight loss. She denies sick contacts, travel history, camping history, or eating at any unusual locations. She is unsure if there is an association with milk products. Her temperature is 36.7ºC (98ºF), blood pressure is 100/70 mm Hg, pulse is 73/min, and respirations are 13/min. Her abdomen is soft, nontender, and nondistended with normal bowel sounds. Which of the following is the most likely etiology of her diarrhea? A. Enteroinvasive E. coli B. Irritable bowel syndrome C. Lactose intolerance D. Pseudomembranous colitis E. Zollinger-Ellison syndrome

C. Lactose intolerance This patient is describing lactose intolerance. There are several clues to the diagnosis and reasons the other choices are incorrect. Lactose intolerance is very common. It is an osmotic diarrhea, which means that undigested food causes the symptoms. Resolution with fasting is another sign that the diarrhea is osmotic. The only answer choice listed that is osmotic is lactose intolerance. In addition, the lack of red flag symptoms such as significant abdominal pain, weight changes, or bloody stools leads us to believe that this is not a more serious condition. Because ingestion of lactose-containing food is the precipitating cause, fasting will improve these symptoms.

Which of the following hereditable gastrointestinal (GI) polyposis syndromes has associated endometrial and ovarian tumors accompanying the polyps? A. Familial adenomatous polyposis B. Gardner syndrome C. Lynch syndrome (familial nonpolyposis syndrome) D. Peutz-Jeghers syndrome E. Juvenile polyposis

C. Lynch syndrome (familial nonpolyposis syndrome) Lynch syndrome is a familial syndrome with a 50% risk of colon carcinoma, which is associated with other primary cancers including endometrial. This nonpolyposis syndrome is due to defective DNA mismatch repair.

A 28-year-old woman has significantly symptomatic peptic ulcer disease. Extensive medical management, including eradication of Helicobacter pylori, fails to heal her ulcers. Endoscopy shows several duodenal ulcers in the first and second portions of the duodenum. She also complains of watery diarrhea. Which of the following is the most appropriate next step in management? A. Biopsy of the duodenal ulcers B. Culture of the watery stools C. Measurement of serum gastrin D. Repetition of the H. pylori eradication every 2 months E. Replenishment of the normal gut flora

C. Measurement of serum gastrin Virulent and extensive peptic ulcer disease should trigger a workup for gastrinoma. The presence of watery diarrhea actually adds to our suspicion that a gastrinoma must be present.

Which of the following diagnostic studies is recommended to perform in the setting of a patient suspected of having chronic mesenteric ischemia in order to confirm the diagnosis? A. CT of the abdomen with oral and IV contrast B. CT of the abdomen without contrast C. Mesenteric arteriography D. Plain upright abdominal film E. MRA of the abdomen

C. Mesenteric arteriography Mesenteric arteriography used to confirm chronic mesenteric ischemia. This study is able to identify the vascular flow pattern to the mesenteric artery and can be used to ascertain the underlying cause for this patient's ischemic pattern such as thrombosis, stenosis, or embolism. Mesenteric arteriography is still considered the definitive test for identifying mesenteric ischemia.

Which of the following laboratory abnormalities is most likely to be seen in the setting of severe diarrhea? A. Hyponatremia B. Hypoglycemia C. Metabolic acidosis with hypokalemia D. Respiratory alkalosis E. Low hydrogen ion concentrations

C. Metabolic acidosis with hypokalemia Severe diarrhea will cause loss of potassium and bicarbonate along with the retention of hydrogen ions, leading to metabolic acidosis and hypokalemia.

A male homosexual is seen in the office with complaints of fever, right upper quadrant pain, nausea/vomiting, hepatomegaly, and diarrhea. Stool antigen test for Entamoeba histolytica is positive. What is the treatment of choice? A. Liver transplantation B. Clindamycin C. Metronidazole D. Ciprofloxacin E. Albendazole (Albenza)

C. Metronidazole Metronidazole is first-line treatment for eradication of amebic liver abscess.

A patient is hospitalized with Clostridium difficile colitis. The patient is not toxic in appearance, but he is having multiple bouts of diarrhea. In addition to fluid resuscitation, which of the following is the initial treatment of choice? A. Oral vancomycin B. IV vancomycin C. Oral metronidazole (Flagyl) D. Clindamycin E. Doxycycline

C. Oral metronidazole (Flagyl) Oral metronidazole is considered the initial best treatment for patients with C. difficile colitis. Fidaxomicin is also considered to be first-line therapy for this condition for patients who have mild to moderate C. difficile infection. This agent should not be used in a patient who is pregnant.

A 63-year-old man in apparent good health is convinced by magazine articles and TV programs that he will live longer if he takes one aspirin tablet every day. After about 3 weeks of doing so (325 mg/day), he begins to notice bright red blood on the toilet paper when he wipes after a bowel movement. This does not occur every time, but only when, for whatever reason, he has to strain more than usual. He has never had any discomfort referable to hemorrhoids or any bowel pathology that he is aware of. Anoscopy and digital rectal examination show external and internal hemorrhoids, none of which are bleeding at the time of the examination. Which of the following is the most appropriate next step in management? A. Discontinue the aspirin B. Perform rubber-band ligation of the internal hemorrhoids C. Perform 60-cm flexible proctosigmoidoscopy D. Recommend stool softeners E. Reduce the aspirin dosage

C. Perform 60-cm flexible proctosigmoidoscopy Normal colonic mucosa does not bleed in response to anticoagulation, but lesions with a predisposition to do so may be more likely to become symptomatic at such time. The bleeding lesion in this case may indeed be the internal hemorrhoids that were diagnosed, but a malignant source has to be ruled out before making such assumption. At the very least, the patient needs a 60-cm examination with a flexible endoscope. If one of the answer choices was colonoscopy, that would be the next best step, because this allows visualization of the entire colon. Because the patient is older than age 50 years, a colonoscopy is warranted as part of the evaluation for potential colon cancer.

A 15-year-old girl comes to the emergency department because of the sudden onset of profuse watery diarrhea. The girl was previously healthy. Her only medications are topical benzoyl peroxide and oral clindamycin for acne vulgaris. Physical examination reveals a slightly distended abdomen that is diffusely tender. Her temperature is 38.1°C (100.5°F). She has not been exposed to any uncooked meat or eaten unusual foods. Which of the following is the most likely diagnosis? A. Gastroenteritis B. Irritable bowel syndrome C. Pseudomembranous enterocolitis D. Salmonella infection E. Ulcerative colitis

C. Pseudomembranous enterocolitis Pseudomembranous enterocolitis is caused by the toxins produced by Clostridium difficile. It occurs in some patients after treatment with antibiotics (especially clindamycin, cephalosporins, and amoxicillin). Patients develop fever and abdominal pain with diarrhea containing leukocytes and blood.

Which of the following tests for Helicobacter pylori infection is useful to perform for patients who have had exposure to H. pylori but is not useful as a test for cure demonstrating eradication of this organism? A. Rapid urease test B. Histology with staining C. Serology testing D. Urea breath testing E. Stool antigen testing

C. Serology testing Serology for H. pylori assesses whether infection is recent (IgM antibodies) or remote (IgG antibodies), but this test will not provide information about eradication following treatment, since IgG measurements will continue to be positive even in the setting of successful eradication.

A patient is admitted to the general hospital floor. The patient complained of skin flushing, sweating, wheezing, and abdominal pain. If carcinoid syndrome is suspected, which of the following neurotransmitters is secreted with this condition? A. Dopamine B. Epinephrine C. Serotonin D. Norepinephrine E. Prolactin

C. Serotonin Carcinoid tumors secrete serotonin, and this is followed with 5HIAA levels.

Which of the following studies is indicated to perform for a hospitalized patient suspected of having acute mesenteric ischemia? A. Blood urea nitrogen (BUN) to creatinine ratio B. Absolute lymphocyte count C. Serum lactate level D. Procalcitonin level E. SED rate

C. Serum lactate level Serum lactate level should be ordered with suspected acute mesenteric ischemia, as these levels will reflect the degree of acidosis and aid in determining the severity of illness.

An 8-year-old girl is brought to the office because she has been complaining of frequent abdominal pain. The episodes occur every several days, show no particular pattern, and resolve without treatment within 10-15 minutes. During the episodes, the child has to lie down and bring her legs up to her chest for relief. The parents state that she started having this problem approximately a year ago, and they believed it was related to her diet. After trying to restrict various kinds of food with no effect, they decided to have her examined. Her bowel movements are regular. She has no significant past medical history and takes no medication on a regular basis. She has an older brother who is in excellent health. The mother tells you that she loves school and is a very good student. She is very popular among her friends and has no problems in social relationships. Of interest, they immigrated to the United States 3 years earlier and have moved 3 times since then for work. She did not seem to have any trouble adjusting to that situation. They will be moving to another city again in 6 months. On physical examination the girl is in no acute distress. She is in the 90th percentile for height and weight. Vital signs are within normal limits, as is a complete blood count with white blood cell differential. Her abdomen is not distended and is soft and nontender to palpation. Plain abdominal films show no abnormalities. Which of the following descriptions/interventions should be pursued? A. A computerized tomography scan of the abdomen is indicated B. A trial of oral antibiotics is warranted C. She is likely to continue having episodes of abdominal pain even as an adult D. She may have Hirschsprung's disease and a biopsy of the colon is indicated E. A barium enema is indicated at this time

C. She is likely to continue having episodes of abdominal pain even as an adult This patient has functional recurrent abdominal pain and it is highly likely that she will continue to have episodes of abdominal pain even as an adult. Patients who have functional recurrent abdominal pain present with nonspecific symptoms. It is difficult to obtain a pattern of occurrence of the pain. Stressors such as school or examinations may produce a predictable pattern. Social factors, such as relocating, family illness, and sibling rivalry may contribute to the abdominal pain. Medically unexplained headache or abdominal pain occurs at least once a week in 10 to 30% of children and adolescents in the United States. Seventy percent of children will continue to experience abdominal pain into adulthood, although the symptoms may not impair their physical activity to the same degree. Up to 30% will develop additional complaints, such as headaches. Parents may have suffered from abdominal pain.

A patient with gastric cancer has metastatic disease signified by a palpable lymph node in the periumbilical area. What is this palpable lymph node known as? A. Krukenberg tumor B. Blumer shelf tumor C. Sister Mary Joseph nodule D. Virchow node E. Irish node

C. Sister Mary Joseph nodule A Sister Mary Joseph nodule is a palpable periumbilical lymph node seen in the setting of metastatic gastric cancer. Krukenberg tumor (choice A) is gastric cancer that has metastasized to the ovary. A Blumer shelf tumor (choice B) is also a sign of gastric cancer spreading to the ovary. Blumer shelf is a palpable mass felt in the cul de sac. A Virchow node (choice D) is a palpable left supraclavicular lymph node typically caused by metastatic gastric cancer. An Irish node (choice E) is a palpable left axillary lymph node that can occur in the setting of metastatic gastric cancer.

A 65-year-old patient with long-standing alcoholism presents to the office with increasing amounts of ascetic fluid. In addition to furosemide (Lasix), which of the following medications is recommended for treatment of ascites? A. Chlorthalidone B. Hydrochlorothiazide C. Spironolactone D. Indapamide (Lozol) E. Chlorothiazide (Diuril)

C. Spironolactone Spironolactone is a potassium-sparing diuretic that acts as a direct antagonist to aldosterone and should be added to the treatment regimen.

A patient with long-standing alcohol-related cirrhosis and ascites is admitted to the hospital with a high fever and mental status change. Examination reveals abdominal pain, vomiting, and fever. Which of the following conditions is most likely? A. Hepatic encephalopathy B. Hepatorenal syndrome C. Spontaneous bacterial peritonitis D. Decompensated cirrhosis E. Portal hypertension

C. Spontaneous bacterial peritonitis Spontaneous bacterial peritonitis causes fever and mental status changes in patients with ascites. A high index of suspicion should be held for patients who have ascites and develop these changes, because sepsis can progress if this is not treated.

A patient presents to the emergency department with acute abdominal pain. Which of the following descriptions of the patient's pain would be consistent with a diagnosis of diverticulitis? A. Burning substernal pain after meals B. Severe, diffuse ache in the periumbilical region C. Steady ache in the left lower quadrant with referral to the back D. Steady, boring epigastric pain with referral to the back E. Sudden, severe pain in the lower quadrant with referral to the flank

C. Steady ache in the left lower quadrant with referral to the back Diverticulitis commonly produces a steady, aching pain, localized to the left lower quadrant of the abdomen, with referral to the back in some cases. In addition, a tubular mass (caused by inflammation) may be appreciated on abdominal examination, and the patient may be febrile with an increased white count. Remember, if a patient presents with symptoms and signs similar to appendicitis, but the complaints concern the left rather than the right side of the abdomen, think of diverticulitis as a likely diagnosis. Also, take note that diverticulosis, not diverticulitis, is the most common cause of massive lower gastrointestinal bleeding in adults.

A patient was diagnosed with esophageal cancer, and staging is going to be done prior to the esophagectomy. Which of the following clinical interventions is most commonly used in order to judge the depth of the tumor for performing staging of this localized cancer? A. CT of the chest B. MRI of the chest C. Transesophageal ultrasound D. Plain chest film E. Positron-emission topography (PET) scan

C. Transesophageal ultrasound Transesophageal ultrasound is used to examine the depth of the invasion of the esophageal tumor and is the most commonly used staging procedure for esophageal cancer. Because the esophagus is not covered by fascia, an esophageal tumor may not be confined and can easily spread through the esophageal muscle.

Which of the following is a known risk factor for the development of primary sclerosing cholangitis? A. Infection with Escherichia coli B. Porcelain gallbladder C. Ulcerative colitis D. Crohn disease E. Diverticulitis

C. Ulcerative colitis Ulcerative colitis is a known risk factor for the development of primary sclerosing cholangitis.

Which of the following diarrheal illnesses is classically associated with rice-water stools, abdominal pain, and vomiting? A. Enterotoxigenic E. coli B. E. coli 0157:H7 C. Vibrio cholerae D. Staphylococcal food poisoning E. Salmonella

C. Vibrio cholerae V. cholerae is associated with rice-water stools and abdominal pain with vomiting. It is typically acquired through water contamination. Some serotypes of this pathogen cause such significant diarrhea with resultant dehydration that death can occur within 12 hours of symptoms.

154. Which of the following types of polyps has the greatest chance of undergoing malignant transformation? A. Tubular B. Tubulovillous C. Villous D. Hamartomas E. Hyperplastic

C. Villous The correct answer is C. Villous adenomas have the greatest chance for malignant transformation.

Which of the following types of polyps has the greatest chance of undergoing malignant transformation? A. Tubular B. Tubulovillous C. Villous D. Hamartomas E. Hyperplastic

C. Villous Villous adenomas have the greatest chance for malignant transformation.

A patient presents to the office with complaints related to diarrhea. She is being seen in follow-up after several studies were ordered. The patient continues to have diarrhea with every bowel movement. There is no blood in her stool, and she does not appear to be toxic. A stool for fecal leukocyte test is negative. Which of the following is the most probable underlying cause for this presentation? A. Campylobacter B. Salmonella C. Viral cause D. Entamoeba histolytica E. Shigella

C. Viral cause Stool studies that are negative for fecal leukocytes along with absence of blood in the stool will not typically show any bacterial organisms.

A patient with a history of hypertension calls his health care provider's office for advice. He has had long-standing heartburn and recently consulted with a gastroenterologist. He underwent an endoscopy and was told that "Barrett mucosa" was found by biopsy. The patient has read in the newspaper that people with this condition will probably develop esophageal cancer. Which of the following is the most appropriate response to this concern? A. "Your concerns are ungrounded." B. "It is foolish to worry because this type of cancer is unlikely to develop and would occur many years later." C. "You should chew food very carefully to prevent the possibility of a mechanical obstruction." D. "Only a small minority of patients with Barrett's esophagus will develop cancer, but you should undergo endoscopic surveillance." E. "You should consult with an oncologist regarding esophageal cancer prevention strategies."

D. "Only a small minority of patients with Barrett's esophagus will develop cancer, but you should undergo endoscopic surveillance." Barrett's esophagus may occur in a small number of patients who have gastroesophageal reflux disease (GERD). This condition is a metaplasia of the normal squamous mucosa of the esophagus to a columnar (glandular) type of epithelium, and is usually seen after repeated acid exposure to the distal esophagus. Tobacco and alcohol use are also thought to contribute to the process but not to the same extent that they contribute to squamous cell esophageal cancer. This condition is most commonly seen in middle-aged Caucasian men who have a long-standing history of reflux.

A 20-year-old man is brought to the emergency department 3 hours after ingesting 50 tablets of 325-mg acetaminophen in a suicide attempt. He has nausea and vomiting, but no other abnormalities on physical examination. His temperature is 37ºC (98.6ºF), blood pressure 135/80 mm Hg, pulse 100/min, and respirations 20/min. Serum acetaminophen concentration is within the range of probable toxicity but serum transaminases and other hepatic markers are normal. Which of the following is the most appropriate next step in management? A. Activated charcoal treatment B. Acetylcysteine treatment C. Penicillamine treatment D. Activated charcoal and acetylcysteine treatment E. Gastric lavage

D. Activated charcoal and acetylcysteine treatment Acetaminophen overdose may result in massive hepatocellular necrosis and fulminant hepatic failure. Liver metabolism of acetaminophen by P-450 cytochrome produces a highly toxic intermediate, which depletes glutathione stores, resulting in accumulation of free radicals. In the first few hours after ingestion, the patient may have nausea and vomiting, but no signs of liver failure. Biochemical evidence of hepatic damage, namely elevation of transaminases, begins 24-48 hours after ingestion. Therapy, therefore, should be started before hepatic transaminases begin to increase. The likelihood of hepatic toxicity is based on a combination of serum acetaminophen levels and time elapsed from ingestion. Nomograms have been developed to determine the likelihood that a specific level of acetaminophen at a given time following ingestion will produce hepatotoxicity. Within 4 hours following ingestion, the initial treatment should include activated charcoal (to adsorb residual drug in the intestine) and N-acetylcysteine. Treatment with N-acetylcysteine is most effective if started within 10 hours after ingestion. The FDA-approved recommendation is to continue oral treatment for 72 hours.

The Charcot triad is seen in which of the following clinical conditions? A. Acute pancreatitis B. Acute common bile duct stone C. Acute cholecystitis D. Acute cholangitis E. Mesenteric thrombosis

D. Acute cholangitis The Charcot triad consists of right upper quadrant pain, fever, and jaundice, which occurs in the setting of acute cholangitis. Suppurative cholangitis can lead to the Reynold pentad, which adds the symptoms of confusion and hypotension to the original three symptoms.

The Charcot triad becomes the Reynold pentad when which of the following signs or symptoms occur? A. Fever B. Right upper quadrant pain C. Jaundice D. Altered mental status E. Anuria

D. Altered mental status TheReynold pentad adds altered mental state and septic shock to the Charcot triad of right upper quadrant pain, fever, and jaundice.

A 50-year-old female is seen in the office with complaints of pruritus that is not responsive to antihistamines and topical skin lotions. Laboratory findings include an increase in the serum alkaline phosphatase level, along with increased serum bilirubin and cholesterol. If primary biliary cholangitis is suspected, which of the following additional laboratory tests is indicated? A. Serum iron level B. Prothrombin gene analysis C. Anti-smooth muscle antibody D. Antimitochondrial antibody E. Anti-liver/kidney microsomal antibody

D. Antimitochondrial antibody Antimitochondrial antibodies are seen in primary biliary cholangitis.

A patient presents with complaints of a tearing type of anal pain following passage of a large bowel movement. The patient complained of this pain when a large, dry stool was passed. He noticed that there was a small amount of blood present on the tissue when wiping. He continued to have a burning type of pain following evacuation of the stool. He is dreading having another large bowel movement. Which of the following treatments is indicated for taking care of this significant pain? A. Sitz baths B. Application of witch hazel compresses C. Thorough cleansing following each bowel movements D. Application of nitroglycerin ointment E. Application of topical podophyllin

D. Application of nitroglycerin ointment The most effective treatments for anal fissure management are application of nitroglycerin ointment to the affected area as this medication causes relaxation of the anal canal, which relieves pain, or injection of botulinum toxin into the anal sphincter on each side of the fissure, which stops the spasm of this structure, thereby relieving the pain.

Which of the following is a known risk factor for the development of adenocarcinoma of the esophagus? A. Chronic ingestion of extremely hot tea B. Smoking opiates C. Smoking cigarettes D. Barrett esophagus E. Comorbid melanoma diagnosis

D. Barrett esophagus Barrett esophagus, chronic gastroesophageal reflux disease, obesity, and smoking are risk factors for the development of adenocarcinoma of the esophagus.

A patient is seen in the office with complaints of nocturnal epigastric pain and gastric pain that is relieved by eating. Stool antigen testing is positive for Helicobacter pylori. Quadruple therapy for this patient includes all of the following medications EXCEPT: A. Omeprazole B. Bismuth subsalicylate C. Metronidazole D. Clarithromycin E. Tetracycline

D. Clarithromycin Quadruple therapy for H. pylori eradication includes metronidazole and tetracycline as antibiotics but not clarithromycin. There is a great deal of resistance to clarithromycin, which has caused this agent to drop from quadruple therapy.

A 54-year-old man comes to the health care provider for a periodic health examination. His family history is significant for his mother who died of a cerebrovascular accident at age 72, his father who died of a myocardial infarction at age 68, and his brother who developed sigmoid cancer at age 60. The patient is on no medications except for aspirin, 81 mg daily. Physical examination is unremarkable. The patient asks for a recommendation regarding current cancer screening. Which of the following is the most appropriate screening test for this patient? A. Annual digital rectal examination and fecal occult blood testing B. Flexible sigmoidoscopy C. Flexible sigmoidoscopy and barium enema D. Colonoscopy E. Genetic testing for the p53 gene

D. Colonoscopy Any patient who has a first-degree relative who has developed an adenoma or colorectal cancer should undergo colonoscopy for screening at age 50, or 10 years before the relative developed the adenoma or carcinoma, whichever comes first. This patient has a brother who had colon cancer at age 60; therefore, a full colonoscopy is warranted. Screening recommendations also recommend screening for colon cancer beginning at age 50 for normal-risk individuals. Although there are various opinions regarding appropriate screening in the "average risk individual," there is a consensus that full colonoscopy is required in patients who have an increased risk (e.g., first-degree relative with a positive history).

A 70-year-old patient is admitted to the hospital with anemia related to ongoing gastrointestinal (GI) bleeding. He complained of hematochezia, and a rectal examination confirmed blood in the stool. After ensuring that the patient is stable, which of the following is the next step in his evaluation? A. Plain film of the abdomen B. Upper endoscopy C. Push procedure D. Colonoscopy E. Anoscopy

D. Colonoscopy Colonoscopy is the first procedure indicated for patients who have hematochezia, as colon cancer is the primary consideration in this group.

Which of the following causes of acute diarrhea are immunocompromised patients at highest risk for acquiring because of their immunocompromised state? A. E. coli 0157-H7 B. Salmonella C. Clostridium perfringens D. Cryptosporidium E. Giardia lamblia

D. Cryptosporidium Immunocompromised patients are at highest risk for diarrhea due to Cytomegalovirus, Cryptosporidium, Mycobacterium avium-intracellulare, and Cyclospora.

A patient with long-standing alcohol-related cirrhosis and ascites is admitted to the hospital with a high fever and mental status change. Examination reveals abdominal pain, vomiting, and fever. Which of the following pathogens is most likely the cause of this presentation? A. Pseudomonas B. Streptococcus pneumoniae C. Staphylococcus aureus D. Escherichia coli E. Klebsiella

D. Escherichia coli E. coli, a gram-negative rod that is part of the enteric bowel bacteria, is the most common cause for spontaneous bacterial peritonitis.

A previously healthy 7-year-old girl comes to the office with complaints of episodic abdominal pain over the past several months. The pain is periumbilical and sharp but does not wake her from sleep and rarely interferes with play. She has no fever, joint complaints, or constipation or diarrhea. Growth and development have been normal. Physical examination is within normal limits. Which of the following is the most likely diagnosis? A. Acute appendicitis B. Acute cholecystitis C. Crohn's disease D. Functional abdominal pain E. Irritable bowel syndrome

D. Functional abdominal pain Functional abdominal pain is pain that lasts for more than 3 months and rarely interferes with normal activity. The pain is periumbilical and often hard to describe. The pain typically does not awaken patients from sleep or interfere with pleasant activities. The pain is real and is the result of the regulation of gastrointestinal motility in response to either psychologic or physical stress.

Which of the following conditions is the most common cause of ascending cholangitis? A. Cholelithiasis B. Pancreatic pseudocyst C. Klatskin tumor D. Gallstone in the ampulla of Vater E. Pancreatic abscess

D. Gallstone in the ampulla of Vater A gallstone in the ampulla of Vater is the most common cause of ascending cholangitis.

A patient is admitted to the hospital with severe right upper quadrant abdominal pain that radiates into the back and right scapula. An ultrasound of the right upper quadrant is normal. The patient is still suspected of having acute cholecystitis but lack of a positive ultrasound finding prohibits surgery. Which of the following diagnostic tests is indicated for further assessment for acute cholecystitis? A. CT of the abdomen B. MRI of the abdomen C. Angiogram of the abdomen D. HIDA scan E. ERCP

D. HIDA scan The health care provider should perform a hepatobiliary iminodiacetic acid (HIDA) scan if ultrasound is normal. In this setting, acute cholecystitis will prevent visualization of the gallbladder in 4 hours, confirming a diagnosis of acute cholecystitis.

A patient with esophageal varices had failed clinical intervention to prevent further bleeding. The patient undergoes transjugular portosystemic shunt (TIPS) surgery in order to lower portal pressures. Which of the following is the most concerning complication for the TIPS procedure? A. Dumping syndrome B. Diabetes insipidus C. Hepatorenal syndrome D. Hepatic encephalopathy E. Spontaneous bacterial peritonitis

D. Hepatic encephalopathy Hepatic encephalopathy, shunt stenosis or occlusion, and infection are complications of this procedure.

A patient is admitted to the hospital with severe right upper quadrant (RUQ) abdominal pain that radiates into the back and right scapula. Although an ultrasound of the RUQ is normal, the patient is nevertheless suspected of having acute cholecystitis. Which of the following diagnostic tests is indicated for further assessment for acute cholecystitis? A. CT of the abdomen B. MRI of the abdomen C. Angiogram of the abdomen D. Hepatobiliary iminodiacetic acid (HIDA) scan E. Endoscopic retrograde cholangiopancreatography (ERCP)

D. Hepatobiliary iminodiacetic acid (HIDA) scan Perform HIDA scan if ultrasound is normal. Patients with acute cholecystitis will have failure to visualize the gallbladder in 4 hours due to cystic duct obstruction.

Which of the following metabolic conditions is most likely to cause constipation? A. Addison disease B. Diabetes C. Short bowel syndrome D. Hypothyroidism E. Gastrinoma

D. Hypothyroidism Hypothyroidism can cause constipation. Hypothyroidism tends to cause decreased metabolic rate, which can translate to slower gastrointestinal transit times and constipation.

A 57-year-old woman with a 30-year history of alcoholism and liver disease visits her health care provider complaining of abdominal swelling and shortness of breath. The health care provider determines that she has severe ascites. Which of the following factors contributes to the accumulation of fluid in the abdominal cavity? A. Decreased plasma epinephrine and norepinephrine B. Decreased plasma volume C. Increased hepatic lymph flow D. Increased hydrostatic pressure in splanchnic capillary beds E. Increased natriuresis

D. Increased hydrostatic pressure in splanchnic capillary beds Ascites often occurs in patients who have cirrhosis and other forms of severe liver disease and is usually noticed by the patient because of abdominal swelling. Shortness of breath may occur because the diaphragm is elevated when the accumulation of fluid becomes more pronounced. A number of factors contribute to accumulation of fluid in the abdominal cavity. Portal hypertension plays an important role in the production of ascites by raising capillary hydrostatic pressure within the splanchnic bed.

Which of the following is a known cause for a false negative Hemoccult study of the stool? A. Ingestion of red meat B. Ingestion of red Kool Aid C. Ingestion of iron D. Ingestion of vitamin C E. Ingestion of bismuth salicylate

D. Ingestion of vitamin C Vitamin C ingestion and intermittent gastrointestinal bleeding can cause false negative tests for occult blood.

A 69-year-old woman comes to her health care provider's office complaining of 1 week of crampy lower abdominal pain and bloody diarrhea. She had previously been followed for symptoms of stable exertional angina and hypertension. She had an uncomplicated myocardial infarction 3 years earlier. Her symptoms began 1 week ago with mild postprandial abdominal cramping followed by diarrhea, which became bloody after 2 days. She has not traveled recently and is a retired librarian. Her temperature is 38.6ºC (101.4ºF), blood pressure 120/84 mm Hg, and pulse 96/min. She has moderate tenderness to palpation of the left lower quadrant. Rectal examination reveals bloody stool and no masses. Which of the following is the most likely diagnosis? A. Arteriovenous malformation B. Diverticulitis C. Diverticulosis D. Ischemic colitis E. Ulcerative colitis

D. Ischemic colitis This elderly woman who has a history of atherosclerotic vascular disease as demonstrated by a history of a myocardial infarction and exertional angina has developed colitis symptoms, as demonstrated by the left lower quadrant pain and bloody diarrhea. This is typically caused by ischemia of small branches of the inferior mesenteric artery. The bleeding is caused by poor blood flow to the bowel, which leads to mucosal sloughing that can cause blood loss into the bowel lumen. The diagnosis is suspected clinically and generally confirmed with a flexible sigmoidoscopy, because many cases involve the rectosigmoid region.

A 23-year-old woman seeks help for exquisite pain with defecation and blood streaks on the outside of her stools, which she has been having for several weeks. Because of the pain, she has avoided having bowel movements, and when she finally did, the stools were hard and the perirectal pain was even more painful. When seen by the healthcare provider, the patient has no fever or leukocytosis. Physical examination has to be done under sedation, because the patient was so afraid of the pain that she initially refused even inspection of the area. The examination confirms the suspected diagnosis, and she is placed on stool softeners and appropriate topical agents, but without success. She is now willing to undergo more aggressive treatment. Which of the following is the most appropriate next step? A. Excision of the lesion B. Fistulotomy C. Incision and drainage D. Lateral internal sphincterotomy E. Rubber band ligation

D. Lateral internal sphincterotomy The clinical picture is classic for anal fissure, which is a split in the skin of the anal canal. Patients with this condition present with pain with defecation along with rectal bleeding from the area of the split. It commonly occurs in young to middle age persons. Pain is described as severe and this can adversely affect quality of life. If spontaneous healing does not occur, surgical intervention with lateral sphincterotomy can be performed.

A 58-year-old man with known hepatitis C and cirrhosis complains of worsening fatigue and confusion over the past 5 days. He has been admitted 3 times in the past 4 months for variceal bleeding and has had ascites that has been refractory to high-dose oral diuretic use. He also reports that over the past 48 hours he has had a declining urinary output. On physical examination, he is gaunt and jaundiced. He has tense ascites and a liver span of 7 cm in the midclavicular line. Laboratory results reveal a white blood cell count of 4,600/mm3 (normal 5,000-10,000/mm3), hemoglobin 9.4 g/dL (normal 13.8-17.2 g/dL), and hematocrit 29% (normal 41-50%). His electrolytes reveal BUN 34 mg/dL (normal 7-20 mg/dL) and creatinine 3.1 mg/dL (normal in men 0.8-1.4 mg/dL). Urinary sodium is less than 10 mEq/L (normal spot sodium >20 mEq/L). Which of the following is the most appropriate treatment for his elevated BUN and creatinine? A. Hemodialysis B. Kidney transplantation C. Large volume paracentesis D. Liver transplantation E. Mesocaval shunt

D. Liver transplantation This patient who has well advanced cirrhosis and portal hypertension has developed the onset of renal insufficiency consistent with hepatorenal syndrome. This occurs during the end stages of cirrhosis and is characterized by diminished urine output and low urinary sodium. In the setting of end-stage liver disease, renal vasoconstriction occurs, and the distal convoluted tubule responds by conserving sodium. Unless the renal function is allowed to deteriorate further, liver transplantation will reverse this vasoconstriction and kidney function will return to normal.

A patient is admitted to the hospital with suspected acute diverticulitis. Which of the following signs or symptoms would be unusual for patients who have acute diverticulitis? A. Altered bowel habits (constipation or diarrhea) B. Left lower quadrant pain C. Fever D. Lower gastrointestinal bleeding E. Leukocytosis

D. Lower gastrointestinal bleeding Lower gastrointestinal bleeding is unusual in the setting of acute diverticulitis, whereas significant left lower quadrant pain is typically the presenting symptom. Pain is the rule with diverticulitis.

An 82-year-old woman is brought to the emergency department from the local nursing home because of complaints of severe constipation. She has not moved her bowels in 5 days and today began complaining of lower abdominal discomfort and distention. She takes hydrochlorothiazide and acetaminophen with codeine for severe arthritic pain in both hips. Over the past week she has been bedridden because of the severe pain. She is afebrile. Abdominal examination shows mid- and lower-abdominal distention with mild tenderness. Bowel sounds are normoactive. Rectal examination demonstrates that the rectal vault is filled with hard stool. Which of the following is the most appropriate next step in management? A. Barium enema B. Colonoscopy C. Flexible sigmoidoscopy D. Manual disimpaction E. Passage of a nasogastric tube

D. Manual disimpaction This elderly woman, who has been bedridden and on a narcotic analgesic, has become severely obstipated. This multifactorial condition is caused by her immobilization, bowel hypomotility secondary to the codeine, and possible mild dehydration from her diuretic. Treatment is manual disimpaction, which will promptly "open the flood gates" and yield immediate relief for the patient.

Which of the following patient populations is most likely to acquire primary biliary cholangitis? A. Alcoholics of either gender B. Alcoholic men C. Alcoholic women D. Middle-aged women E. IV drug users of either sex

D. Middle-aged women Primary biliary cholangitis is most commonly seen in middle-aged women.

A patient in the hospital is admitted with acute mesenteric ischemia caused by an acute embolism of the superior mesenteric artery. The patient has ongoing symptoms. In addition to resuscitative and supportive measures and empiric antibiotics, which of the following is the initial therapy of choice for this patient? A. Immediate transverse colectomy B. Dopamine infusion C. Norepinephrine infusion D. Papaverine infusion E. Heparin

D. Papaverine infusion Papaverine (vasodilator) infusion is indicated for acute mesenteric thrombosis due to embolism. This treatment option is typically given along with arterial reconstruction or bypass. A bowel resection may be needed if there is gangrenous bowel.

A patient is admitted to the hospital with abdominal complaints. Which of the following patients is at highest risk for developing acute mesenteric ischemia due to nonocclusive mesenteric ischemia? A. Patient with atrial fibrillation B. Patient with peripheral vascular disease C. Patient with hypercoagulable state D. Patient who is critically ill and elderly E. Patient taking oral contraceptives

D. Patient who is critically ill and elderly Critically ill, elderly patients are at highest risk for development of nonocclusive mesenteric ischemia secondary to splanchnic vasoconstriction secondary to low cardiac output.

A patient undergoes a total gastrectomy because of a proximally located gastric cancer. After the surgery, which of the following digestive enzymes will be produced in inadequate amounts? A. Amylase B. Chymotrypsin C. Lipase D. Pepsin E. Trypsin

D. Pepsin Pepsin is secreted by the chief cells of the stomach. Pepsinogen is activated by contact with stomach acid.

A patient undergoes colonoscopy, which reveals multiple hamartomas scattered throughout the gastrointestinal tract. The patient is also noted to have pigmented spots around the lips, oral mucosa, and palmar surfaces. Which of the following is the most likely diagnosis? A. Familial adenomatous polyposis B. Gardner syndrome C. Turcot syndrome D. Peutz-Jeghers syndrome E. Hereditary nonpolyposis colorectal cancer

D. Peutz-Jeghers syndrome Peutz-Jeghers syndrome is associated with mucocutaneous macules on the face and mucosal surfaces along with multiple colonic hamartomatous gastrointestinal tumors.

Which of the following hereditable gastrointestinal (GI) polyposis syndromes has associated hamartomatous polyps of the entire GI tract and mucocutaneous pigmentation accompanying these polyps? A. Familial adenomatous polyposis B. Gardner syndrome C. Lynch syndrome (familial nonpolyposis syndrome) D. Peutz-Jeghers syndrome E. Juvenile polyposis

D. Peutz-Jeghers syndrome Peutz-Jeghers syndrome results in numerous hamartomatous polyps of the entire GI tract with the highest number in the small bowel. These patients are at increased risk for the development of cancer at both GI and non-GI sites.

A 53-year-old woman comes to the health care provider complaining of fatigue over the past 6 months. During this time, she has also developed pruritus and lost 4 pounds. She is not sexually active. On physical examination she is afebrile and has mildly icteric sclera. There are excoriations noted on all 4 extremities and trunk and back. The liver edge is smooth and non-tender and measures 9 cm at the midclavicular line. There is no ascites, splenomegaly, or peripheral edema. Laboratory results reveal a normal complete blood count, normal electrolytes, liver function tests revealing an alkaline phosphatase of 260 U/L (normal <110 U/L), total bilirubin of 3.1 mg/dL (normal <1.9 mg/dL), and normal transaminase levels. Which of the following is the most likely diagnosis? A. Acute cholecystitis B. Acute hepatitis A infection C. Bacterial cholangitis D. Primary biliary cirrhosis E. Primary sclerosing cholangitis

D. Primary biliary cirrhosis This woman has a classic presentation of primary biliary cirrhosis. This condition typically affects middle-aged women and will progress gradually to the point of end-stage liver disease over a number of years. The disease is caused by an autoimmune destruction of intrahepatic bile ducts, and the diagnosis is made by liver biopsy. The serology that should be checked is the antimitochondrial antibody. Primary biliary cirrhosis is often seen in individuals who have other autoimmune diseases, such as pernicious anemia, type 1 diabetes, and Hashimoto thyroiditis.

Various treatments are given for the management of gastrointestinal (GI)-related complaints. What is the mechanism of action for metoclopramide (Reglan) in the management of GI complaints? A. Blocks histamine release B. Blocks histamine receptors C. Stimulates the proton pump to stop making acid D. Promotility agent E. Antacid

D. Promotility agent Metoclopramide, a dopamine agonist, is a promotility agent used in the management of gastroparesis, gastroesophageal reflux disease, and chemotherapy-related and post-operative nausea and vomiting. This agent sensitizes tissues to acetylcholine, which stimulates upper GI tract motility.

A 5-month-old boy is brought to the office because of frequent watery stools and vomiting present for the past 2 days. The child has been in daycare since age 3 months and has had several episodes of fever and runny nose in the meantime, but this is the first time he has had diarrhea. It started approximately 48 hours earlier with the passage of six to seven loose, watery stools with a foul smell. He had also vomited a couple of times and has had fever since the beginning of the symptoms. The parents had called the daycare to see if other children were affected, but learned that no one else had similar symptoms. They had not traveled anywhere since the child was born. On physical examination, the child is in moderate distress, with a temperature of 38.1ºC (100.6ºF), a pulse of 120/min, and respirations of 26/min. He has dry mucosa and his anterior fontanelle is sunken. Skin turgor is also diminished. Palpation of the abdomen does not reveal tenderness. On auscultation, increased peristalsis can be heard. The stool is guaiac-positive for occult blood but shows no visible gross blood on examination. He is given intravenous rehydration in the office and sent home on supportive treatment with lots of fluids and antipyretics as needed. Two days later, the mother calls the office to say that all the symptoms have completely resolved and the child is feeling well and behaving normally. Which of the following was the most likely cause of this patient's illness? A. Clostridium difficile B. Clostridium perfringens C. Enterotoxigenic Escherichia coli D. Rotavirus E. Staphylococcus aureus

D. Rotavirus Diarrhea is defined as an increased stool output, with excess loss of fluid and electrolytes. It can be classified as acute or chronic. Several mechanisms exist that cause diarrhea, of which more than one may be present. Causes of acute and chronic diarrhea are age-dependent. Acute diarrhea is almost always infectious, with gastroenteritis the most common cause in any age group. Food poisoning, systemic infections, parasitic infections, and antibiotics are other causes. Viral agents are the most common cause of acute diarrhea in children, and rotavirus is the most common viral cause of diarrhea in the winter. Enteric adenovirus and Norwalk virus are also common causes of diarrhea. Bacterial causes include E. coli, Salmonella, Shigella, Campylobacter, Yersinia, and Clostridium. Parasitic causes include Entamoeba histolytica, Giardia, and Cryptosporidium. Clinically, rotavirus infection presents with watery diarrhea that can last up to 7 to 10 days. It may be accompanied by 3 to 4 days of vomiting. Fever may be present. The stool may be positive for occult blood. There is no abdominal tenderness associated with rotavirus infection.

A 51-year-old man undergoes a barium enema as a colon cancer screening examination. A 3-mm polyp is found in the ascending colon. It is rounded and smooth and meets all the radiographic criteria of a benign sessile polyp. No other lesions are seen within the colon. Which of the following is the most appropriate management? A. Repeat the barium enema in 1 year B. Schedule a colonoscopy in 1 year C. Schedule a sigmoidoscopy now D. Schedule a colonoscopy now E. Schedule a CT scan to rule out nodal involvement

D. Schedule a colonoscopy now The finding of a sessile mass in the colon, even though it is small and has a benign appearance, mandates a colonoscopy for its removal. This would allow a complete excisional biopsy and histologic review. Although frank carcinoma is unlikely with a mass this small, either a tubular adenoma or a villous adenoma is conceivably present and should be removed both to establish the diagnosis and to prevent the lesion from eventually progressing to cancer. In addition, the structure of the bowel is sufficiently complex that it is not uncommon for other small polyps to be present that were not picked up radiologically. These should also be excised, and their histology reviewed. The need for another diagnostic test to be performed highlights the advantage of performing the colonoscopy as the initial screen rather than barium enema.

Which of the following colonic conditions can be treated with sigmoidoscopy? A. Diverticulitis B. Angiodysplasia C. Diverticulosis D. Sigmoid volvulus E. Cecal volvulus

D. Sigmoid volvulus Sigmoidoscopy can be used to treat sigmoid volvulus acutely, and the patient typically undergoes more definitive therapy in the future to prevent recurrence.

Which of the following causes of infectious diarrhea is most commonly acquired from eating food containing mayonnaise? A. Enterotoxigenic E. coli B. E. coli 0157:H7 C. Vibrio parahaemolyticus D. Staphylococcal food poisoning E. Salmonella

D. Staphylococcal food poisoning Staphylococcal food poisoning is acquired after food is handled by a carrier such as salads, dairy products, produce, milk, and eggs, which would include the ingestion of mayonnaise.

A patient with gastric cancer has metastatic disease to the supraclavicular nodes on the left side. What is this metastatic disease known as? A. Krukenberg tumor B. Blumer shelf tumor C. Sister Mary Joseph nodule D. Virchow node E. Irish node

D. Virchow node A Virchow node is a palpable left supraclavicular lymph node typically caused by metastatic gastric cancer.

You are managing a postoperative patient who underwent an aortic aneurysm resection several days ago. The patient began having melena, and you are called to assess him. Which of the following causes of this gastrointestinal (GI) bleeding should be primarily investigated? A. Duodenal ulcer B. Gastric ulcer C. Arteriovenous (AV) malformation D. Dieulafoy vascular malformation E. Aortoenteric fistula

E. Aortoenteric fistula Aortoenteric fistula causes GI bleeding following aortic aneurysm resection, possibly from pulsation of the vessel against the graph wall, which leads to bleeding.

A patient is seen in the office with rectal bleeding in the form of melena. The patient has noticed intermittent darkening of her stool for the last several months. She is also complaining of not having energy and of dyspnea on exertion. The patient is evaluated with an arteriogram, which shows clusters of small vessels located on the right side of the colon. The patient undergoes colonoscopy, which reveals flat, bright red, fernlike lesions. Which of the following conditions is most likely the cause of this condition? A. Acute diverticulitis B. Acute mesenteric ischemia C. Chronic mesenteric ischemia D. Acute appendicitis E. Arteriovenous malformation

E. Arteriovenous malformation Arteriovenous malformation has vascular ectasias in the right colon leading to lower gastrointestinal bleeding. This condition can be chronic in nature rather than abrupt with slow amounts of blood loss that occur over time.

A 65-year-old man presents to the office with vague abdominal pain that he describes as a dull ache. He has been anorexic and has been losing weight. There is a suggestion of jaundice. All of the following would be known risk factors for this condition EXCEPT: A. Cigarette smoking B. Chronic pancreatitis C. Heavy alcohol use D. Obesity E. Biliary colic

E. Biliary colic Biliary colic is not known risk factor for the development of pancreatic cancer.

195. A patient is evaluated for cholestasis symptoms and found to have bead-like structuring and bead-like dilations of the intrahepatic and extrahepatic ducts. Which of the following treatments would be most likely to help with symptomatic relief of pruritus? A. Ursodiol (Actigall) B. Hyoscyamine (Anaspaz) C. Penicillamine (Cuprimine) D. Lactulose E. Cholestyramine (Questran)

E. Cholestyramine (Questran) The correct answer is E. Patients with primary biliary cholangitis have symptomatic response to cholestyramine for pruritus.

A patient in the hospital is admitted with acute mesenteric ischemia caused by mesenteric venous thrombosis. The patient has ongoing symptoms, but there are no signs of peritonitis. Which of the following is the initial therapy of choice for this patient? A. Immediate transverse colectomy B. Dopamine infusion C. Norepinephrine infusion D. Papaverine infusion E. Heparin

E. Heparin Heparin infusion can be effectively used in the setting of acute mesenteric thrombosis due to venous thrombosis as part of the treatment for patients with this condition. These patients are given appropriate supportive measures and empiric antibiotics along with anticoagulation with heparin.

219. A patient undergoes esophagogastroduodenoscopy, and squamous cell carcinoma of the esophagus is found. All of the following are known risk factors for the development of this condition EXCEPT: A. Tobacco use B. Alcohol use C. Achalasia D. Lye ingestion E. Chronic indigestion/reflux

E. Chronic indigestion/reflux The correct answer is E. Chronic gastrointestinal reflux is a risk factor for adenocarcinoma, not squamous cell carcinoma of the esophagus. Clinical Pearls • In the past, esophageal cancer was primarily squamous cell cancer with the primary risk factors being use of alcohol and tobacco. With the increase in the rate of abdominal obesity, more patients are developing esophageal reflux with an increase in the number of adenocarcinoma cases. • Barrett's esophagus, with the change in the lower esophagus from squamous to columnar cells, puts a patient at risk for the development of adenocarcinoma when these columnar cells undergo dysplasia and then overt cancer. • Barrett's esophagus occurs as a result of these cells in the distal esophagus undergoing changes when they are exposed to gastric acid from reflux. • Patients who are colonized with H. pylori infection who develop GERD are less likely to have Barrett's esophagus as the materials refluxing not being as acid due to the H. pylori colonization.

A 53-year-old woman comes to the health care provider for an annual examination. She has no complaints. She has hypertension, for which she takes a thiazide diuretic, but no other medical problems. Her past gynecologic history is significant for normal annual Pap tests for many years, her last being 2 months ago. A recent mammogram was negative. Heart, lung, breast, abdomen, and pelvic examinations are unremarkable. Which of the following procedures or tests should most likely be performed on this patient? A. Chest radiograph B. Pap test C. Pelvic ultrasound D. Cardiac stress test E. Colonoscopy

E. Colonoscopy Cancer screening should be an essential part of an annual examination. Colorectal cancer is a major cause of serious morbidity and mortality for women in the United States: greater than 50,000 new cases are diagnosed each year, and there are more than 25,000 deaths from colorectal cancer. Screening should begin at age 50 in asymptomatic women who have no significant family history. Screening options for colorectal cancer include home fecal occult blood test (FOBT), flexible sigmoidoscopy, the combination of home FOBT and flexible sigmoidoscopy, colonoscopy, DNA testing of the stool, and double-contrast barium enema.

Which of the following diagnostic evaluations is the preferred clinical intervention to pursue in a patient having a lower gastrointestinal (GI) bleed caused by angiodysplasia of the colon (arteriovenous malformations)? A. Plain abdominal films B. CT scan without contrast C. MRI D. Barium enema E. Colonoscopy

E. Colonoscopy Colonoscopy is the preferred intervention for patients who have lower GI bleeding caused by AV malformation. Colonoscopy can identify and treat angiodysplasia.

All of the following are risk factors for the development of sigmoid volvulus EXCEPT: A. Institutionalized elderly B. Chronic constipation C. Use of antimotility drugs D. Chronic laxative abuse E. Congenital lack of fixation of sigmoid colon

E. Congenital lack of fixation of sigmoid colon Risk factors for sigmoid volvulus include being elderly, being institutionalized, and having chronic constipation with laxative use. It is cecal volvulus that occurs due to lack of fixation of the right colon, and this does not apply to the risks for sigmoid volvulus.

A patient is admitted to the hospital with suspected choledocholithiasis. The patient has a normal ultrasound of the right upper quadrant. Which of the following diagnostic strategies is indicated? A. CT of the abdomen B. MRI of the abdomen C. Angiogram of the abdomen D. Hepatobiliary iminodiacetic acid (HIDA) scan E. Endoscopic retrograde cholangiopancreatography RCP

E. Endoscopic retrograde cholangiopancreatography RCP ERCP or laparoscopic cholecystectomy with exploration of the common duct for removal of the common duct stone is a clinical intervention done in patients who have choledocholithiasis.

A 31-year-old woman smashes her car against a bridge abutment. She sustains multiple injuries, including upper and lower extremity fractures. She is fully awake and alert, and she reports that she was not wearing a seat belt and distinctly remembers hitting her abdomen against the steering wheel. Her blood pressure is 135/75 mm Hg and pulse88/min. Physical examination reveals a rigid and tender abdomen. There is severe tenderness when external pressure is applied to her abdomen and then suddenly released. She has no bowel sounds. Which of the following would be the most appropriate step to pursue for this patient? A. Continued clinical observation B. CT scan of the abdomen C. Sonogram of the abdomen D. Diagnostic peritoneal lavage E. Exploratory laparotomy

E. Exploratory laparotomy The presence of an "acute abdomen," which this woman has, is an indication for exploratory surgery and prompt repair of the injuries (probably affecting hollow viscera) that have produced the signs of peritoneal irritation.

Which of the following is the most likely cause of a patient with cirrhosis to develop macrocytic anemia? A. Blood loss anemia from poor development of clotting factors B. Concurrent hypothyroidism C. Vitamin B12 deficiency state D. Pernicious anemia state E. Folate deficiency

E. Folate deficiency Folate deficiency from long-term alcohol use and a poor nutritional state in cirrhosis is likely to lead to macrocytic anemia.

A 44-year-old woman with a history of recurrent biliary colic presents with 18 hours of very severe right upper quadrant pain, fever, and jaundice. An abdominal ultrasound reveals a markedly dilated common bile duct. Multiple gallstones are seen in the gallbladder. Which of the following would best determine whether there is cystic duct obstruction? A. Abdominal CT scan B. Abdominal MRI C. Abdominal ultrasound with Doppler flow studies D. Endoscopic retrograde cholangiopancreatograph (ERCP) E. HIDA scan

E. HIDA scan This patient has evidence of a prolonged episode of biliary colic, which may progress to acute cholecystitis. An abdominal ultrasound has revealed a markedly dilated common bile duct, as well as gallstones within the gallbladder. Acute cholecystitis is the result of cystic duct obstruction, and this would be best demonstrated by a HIDA scan. A HIDA scan shows visualizes the iminodiacetic acid, which follows the flow of bile (i.e., from the bloodstream into the liver, into the gallbladder, and then through the cystic duct into the common bile duct).

Cholesterol gallstones are seen in all of the following conditions EXCEPT: A. Obesity B. Multiple pregnancies C. Crohn disease D. Ileal resection E. Hemolysis

E. Hemolysis Patients who have chronic hemolysis can develop black or pigmented stones.

258. A patient is seen in the office complaining of pain in the epigastrium, radiating to the back. The pain is described as steady and dull but also severe. Lying down and eating meals make the pain worse. The patient is anorexic at present but did have a great deal of nausea and vomiting prior to this. Examination reveals abdominal distention along with epigastric tenderness. Which of the following electrolyte abnormalities can lead to this presentation? A. Hyponatremia B. Hypokalemia C. Hyperkalemia D. Hypocalcemia E. Hypercalcemia

E. Hypercalcemia The correct answer is E. Significant hypercalcemia can lead to acute pancreatitis, possibly because calcium can be deposited in the pancreatic ducts, and that calcium can also lead to the activation of trypsinogen in the parenchyma of the pancreas.

Which of the following contributes to pancytopenia that is seen in a patient with long-standing cirrhosis of the liver? A. Blood loss anemia B. Folate deficiency C. Vitamin B12 deficiency D. Bone marrow failure E. Hypersplenism

E. Hypersplenism Cirrhotic patients develop hypersplenism, which can lead to removal of cells from the systemic circulation.

A patient is admitted with severe abdominal pain that is due to mesenteric ischemia. Acute mesenteric ischemia can be caused by a variety of processes. Which of the following causes of mesenteric ischemia occurs as a result of non-occlusive disease? A. Antithrombin III deficiency B. Atherosclerosis C. Vasculitis from systemic lupus erythematosus D. Factor V Leiden mutation E. Hypotension

E. Hypotension Nonocclusive causes of mesenteric ischemia include hypotension; heart failure; arrhythmia; and the use of digitalis, which acts as a vasoconstrictor.

Which of the following is a contributing cause of a patient having low-grade mucosal-associated lymphoid tissue (MALT) lymphoma? A. Smoking cigarettes B. Eating highly processed foods containing nitrates C. Eating raw foods D. Blood type A E. Infection with Helicobacter pylori

E. Infection with Helicobacter pylori Low-grade MALT lymphoma is caused by H. pylori infection, and eradication of this infection causes complete remission in 50% of patients.

Hemochromatosis may affect a variety of organs in the body with iron deposition. Which of the following is considered to be the primary organ involved with this iron deposition disease? A. Heart B. Joints C. Skin D. Thyroid E. Liver

E. Liver The liver is the primary organ involved with hemochromatosis, as this organ preferentially absorbs iron once the red blood cells and bone marrow are saturated with it.

A 35-year-old woman complains of increasing weakness and shortness of breath. A complete blood count with differential indicates a megaloblastic anemia. Her vitamin B12 level is found to be low. She is also found to have hypothyroidism and diabetes and she is noted to have gastritis.Which of the following is associated with this illness? A. Antral involvement B. Decreased serum gastrin level C. Helicobacter pylori infection D. Non-steroidal anti-inflammatory drugs (NSAIDs) E. Parietal cell antibody

E. Parietal cell antibody Gastritis may be immunologically mediated when it is caused by pernicious anemia. This autoimmune disease is associated with an elevated level of parietal cell antibody. Acid secretion is thus reduced in the setting of pernicious anemia. As an autoimmune disease, it is commonly associated with other autoimmune conditions such as hypothyroidism and type 1 diabetes.

Patients with acute cholecystitis caused by gallstones typically have their gallbladder removed, but in certain circumstances, it is preferable to dissolve the stones with medication. Which of the following patients with symptomatic gallstones has an indication for using ursodeoxycholic acid (Actigall) as the management for this condition? A. Patients with porcelain gallbladder B. Patients with pericholecystic fluid collection C. Patients with gallbladder gangrene D. Patients with ulcerative colitis E. Patients who are not candidates for surgery

E. Patients who are not candidates for surgery Ursodeoxycholic acid gallstone dissolution therapy is indicated for patients who are not candidates for surgery. This agent needs to be taken on a long-term basis, because the stones will recur whenever the medication is stopped. Some of these patients who need to have ongoing drainage of their gallbladder but who are too sick to benefit from surgery may need to have cholecystostomy with bag drainage over a long period of time while still taking this medication.

A 43-year-old man develops excruciating abdominal pain at 8:23 PM (he looked at his watch when the pain "hit him"). When seen in the emergency department about 30 minutes later, he has a rigid abdomen, lies motionless on the examination table, has no bowel sounds, and is obviously in great pain which he describes as constant and encompassing the entire abdomen. There is very severe pain when deep palpation of the abdomen is attempted in any of the 4 quadrants. The examining hand cannot make much of an indentation, however, because of the impressive muscle guarding. There is also severe rebound tenderness. Radiographs show free air under both diaphragms. Which of the following diagnosis is most likely? A. Acute abdomen, the nature of which cannot yet be defined B. Acute inflammatory process affecting an intra-abdominal viscera C. Acute obstruction of an intra-abdominal viscera D. Ischemic process affecting intra-abdominal organs E. Perforation of the gastrointestinal tract

E. Perforation of the gastrointestinal tract There is no doubt that this patient has an acute abdomen, but we can tell more than that. The sudden onset, generalized extent, and silent abdomen in a man who does not want to move suggest a perforation. In addition, the presence of free air in the peritoneal cavity pinpoints the gastrointestinal tract as the source. We cannot tell whether he perforated a peptic ulcer, blew out a sigmoid diverticulum, or had his bowel perforated by a chicken bone, but there is a hole in his gastrointestinal tract.

A 50-year-old female is seen in the office with complaints of pruritus that is not responsive to antihistamines and topical skin lotions. Laboratory findings include an increase in the serum alkaline phosphatase level, along with increased serum bilirubin and cholesterol. Which of the following is the most likely diagnosis? A. Wilson disease B. Hemochromatosis C. Alpha-1 antitrypsin deficiency D. Alcoholic hepatitis E. Primary biliary cholangitis

E. Primary biliary cholangitis Primary biliary cholangitis has itching along with elevations in alkaline phosphatase, bilirubin, and cholesterol.

Which of the following locations of colorectal cancer is most likely to be associated with the feeling of incomplete emptying of stool? A. Transverse colon B. Right-sided tumors C. Left-sided tumor D. Sigmoid colon E. Rectum

E. Rectum Rectal masses lead to tenesmus and the feeling of incomplete emptying of stool.

Which of the following types of infectious diarrhea is most likely to be acquired by handling or eating domestic fowl or their eggs? A. Enterotoxigenic E. coli B. E. coli 0157:H7 C. Vibrio parahaemolyticus D. Staphylococcal food poisoning E. Salmonella

E. Salmonella Salmonella is typically acquired from poultry and their eggs, or it can be acquired via the fecal-oral route. Salmonella can be acquired from contaminated milk, meats, spices, and eggs.

A 19-year-old college freshman undergoes sigmoidoscopy because of a family history of multiple polyps in his siblings at young ages. His brother underwent total proctocolectomy at age 23, and his sister underwent a total proctocolectomy at age 29, after both were found to have hundreds of colonic adenomas on colonoscopy. Both siblings are alive and well 5 years later and without any other findings of neoplasms. The patient undergoes sigmoidoscopy and is found to have several dozen small colonic polyps within the rectosigmoid area of the colon.Which of the following is the most appropriate next step in management? A. Schedule a repeat sigmoidoscopy in 1 year B. Schedule a colonoscopy in 1 year C. Evaluate the more proximal colon with a barium enema D. Schedule a full colonoscopy now E. Schedule a total proctocolectomy

E. Schedule a total proctocolectomy This patient has two first-degree relatives who have undergone total proctocolectomies for a polyposis syndrome. This most likely represents the familial polyposis coli syndrome in that both siblings had high numbers of colonic adenomas and underwent an appropriate total proctocolectomy. If this patient is found to have several dozen polyps during the flexible sigmoidoscopy, it is evidence that he is expressing the phenotypic trait of the familial adenomatous polyposis (FAP) gene. Rather than wait for any of these polyps to undergo malignant degeneration, he should have a total proctocolectomy in the immediate near future. Once many polyps have been demonstrated, the likelihood of one of these becoming malignant is high.

A 36-year-old man with ulcerative colitis develops pruritus and fatigue. Alkaline phosphatase is elevated. The biliary tree appears beaded on barium radiograph. Which of the following is the most likely diagnosis? A. Acute cholecystitis B. Cholesterolosis C. Chronic cholelithiasis D. Gallstone ileus E. Sclerosing cholangitis

E. Sclerosing cholangitis Young men who have ulcerative colitis are at increased risk for developing primary sclerosing cholangitis (PSC), a chronic cholestatic condition that leads to fibrosis of the bile ducts. This cholestasis leads to an elevation in the alkaline phosphatase level. A classic clue to the diagnosis is a beaded appearance of the biliary tree on barium radiograph.

A 47-year-old man is seen in the primary care office. He has been sick for some time, and the health care providers do not seem to know what is wrong with him. The precipitating event for their visit today was the fact that his sclera turned yellow a few days ago. On further questioning, a history of protracted diarrhea is elicited. For about 3 years the patient has had intermittent episodes of crampy abdominal pain and diarrhea that is often bloody. On one occasion he almost had an appendectomy because of right lower quadrant abdominal pain; surgery was canceled when he improved with observation and a sonogram had been read as negative for acute appendicitis. At this time he has a bilirubin of 3.6 mg/dL (normal <1.9 mg/dL) with 2.9 mg/dL conjugated (normal <0.3 mg/dL), alkaline phosphatase 625 U/L (normal 44-147 u/L), and near-normal transaminases (AST, ALT levels). Sonogram of the right upper quadrant shows normal caliber extrahepatic ducts, no gallstones, and no evidence of liver abscess. Stool cultures are negative. He has never been out of the country. Which of the following is the most likely diagnosis? A. Acute hepatitis B. Amebic abscess of the liver C. Cancer of the head of the pancreas D. Ischemic colitis E. Sclerosing cholangitis associated with inflammatory bowel disease

E. Sclerosing cholangitis associated with inflammatory bowel disease Jaundice with very high alkaline phosphatase is consistent with cholangitis. Cholangitis is typically from obstruction or cholestasis, but sonogram does not show dilated extrahepatic ducts. Normal caliber extrahepatic ducts without gallstones or gallbladder sludge findings are most consistent with sclerosing cholangitis. Furthermore, the current liver findings must be in some way related to his intermittent diarrhea ("always seek a single diagnosis to explain all the findings"), and sclerosing cholangitis is seen in conjunction with inflammatory bowel disease, which in turn is a perfect bet for intermittent, crampy diarrhea in a patient with recurring disease.

A 54-year-old woman complains of severe lower abdominal pain and distention. The symptoms began approximately 24 hours ago, when her abdomen became visibly swollen and she developed nausea and vomiting. She has not moved her bowels over the past 24 hours. Over the past 4 months, she has lost 14 pounds and has noted progressive symptoms of constipation. On several occasions she has noted blood mixed in with her bowel movements, which have become thinner in caliber. She denies any recent travel, use of antibiotics, or fevers. On physical examination she appears acutely uncomfortable and has a temperature of 38.3ºC (100.9ºF). The abdomen is diffusely distended and tender to palpation in the left lower quadrant. There are hyperactive rushing bowel sounds. On rectal examination the stool is brown and guaiac positive. An obstructive series reveals multiple small bowel air fluid levels and a dilated colon proximal to the sigmoid colon. Which of the following is the most likely diagnosis? A. Amebic abscess B. Colon polyp C. Diverticulitis D. Diverticulosis E. Sigmoid carcinoma

E. Sigmoid carcinoma This patient has symptoms of a chronic gastrointestinal process as demonstrated by her weight loss, change in bowel habits, and thinner caliber stools with bleeding. The thinner caliber stools specifically suggest that a mass lesion or luminal narrowing is present in the distal colon since this is where the most formed portion of the stool is present. Of the lesions listed, only colonic cancer would be likely to produce this pattern. In other settings, Crohn's disease and tuberculosis of the colon could produce similar clinical patterns. This patient's change in symptoms over the past 24 hours suggests that she may have developed an acute large bowel obstruction secondary to the colonic mass.

A 55-year-old HIV-positive man has a fungating mass growing out of the anus. He can feel it when he wipes himself after having a bowel movement, though it is not painful. For the past 6 months he has noticed blood on the toilet paper and from time to time there has also been blood coating the outside of the stools. He has lost weight, and he looks emaciated and ill. On physical examination the mass is easily visible. It measures 3.5 cm in diameter, is fixed to surrounding tissues, and appears to grow out of the anal canal. He also has rock-hard, enlarged lymph nodes on both groins, some of them as large as 2 cm in diameter. Which of the following is the most likely diagnosis? A. Adenocarcinoma of the rectum B. Condyloma acuminata of the anus C. External hemorrhoids D. Rectal prolapse E. Squamous cell carcinoma of the anus

E. Squamous cell carcinoma of the anus The entire description is classic for anal cancer, but the best evidence is the presence of metastasis in the inguinal nodes. The mass that is visible is consistent with an anal mass. Anal cancer is most likely squamous cell carcinoma in etiology.

57. Two bewildered parents rush into the emergency department carrying a 3-year-old girl. They tell the health care provider at the door that she ingested multiple acetaminophen pills less than an hour ago. The mother was experiencing a terrible headache over the past 2 days and was taking acetaminophen as a pain reliever. She left the open bottle on her desk to get a glass of water, and when she returned her daughter was putting the tablets in her mouth. Immediately the mother removed whatever pills were still in the child's mouth and checked to insure that all the other medications were safely stored away; when she saw that they were untouched in the medicine cabinet, the family got in the car to rush to the hospital. On route there the girl started vomiting. The mother does not know how many pills her daughter took because it wasn't a full bottle to begin with. At the hospital she takes out the half-empty bottle and spills the contents on the counter. A total of 36 pills is present from an original 100 count, 325 mg of acetaminophen each. The child, crying on the examining table, appears frightened and diaphoretic, and repeats several times that she wants to throw up. Her pulse is 120/min and respirations are 24/min. Which of the following is the most appropriate next step? A. Administration of dextrose and naloxone (Narcan) B. Administration of N-acetylcysteine (Mucomyst, Acetadote) intravenously C. Administration of N-acetylcysteine orally D. Immediate measurement of plasma concentration of acetaminophen E. Supportive care and measurement of plasma concentration of acetaminophen 3 hours later

E. Supportive care and measurement of plasma concentration of acetaminophen 3 hours later The correct answer is E. Acetaminophen is an analgesic and antipyretic that is metabolized in the liver. Acetaminophen poisoning causes an N-acetyl-p-benzoquinoneimine metabolite that produces hepatotoxicity in the absence or depletion of glutathione. In children younger than age 12 the toxic dose is 150 mg/kg. A single ingestion of 7.5 g is considered to be a minimum toxic dose in adolescents and adults. There are 4 stages of acetaminophen poisoning if the patient is left untreated. • Absorption may be delayed 4 hours in an overdose. A plasma level of acetaminophen therefore should be measured when it has been at least 4 hours since the ingestion. Levels drawn before that may not be accurate and may give a false low result. The Rumack-Matthew nomogram is used to plot the level of acetaminophen based on the time elapsed since ingestion and the acetaminophen plasma concentration. N-acetylcysteine is the antidote for acetaminophen poisoning, and it should be administered if the acetaminophen level is equal to or greater than 150 µg/ml. N-acetylcysteine works best if started within 8 hours of ingestion, but it may be given up to 16 hours after ingestion. In cases of severe poisoning, Poison Control may advise N-acetylcysteine be given as late as 24 to 36 hours after ingestion has occurred. In the United States, N-acetylcysteine is given orally. Liver enzymes, bilirubin levels, and prothrombin time should be monitored in patients whose plasma levels of acetaminophen are within the toxic range in the nomogram. In patients who have fulminant hepatic necrosis, death may occur. Severe hepatic damage may necessitate a liver transplant in some patients.

A 22-year-old sociologist returns from 6 months in Jamaica and notes a 6-pound weight loss during her first month back home. She reports that her appetite is normal. She has developed symptoms of mild abdominal cramping and bloating after meals and frequent greasy bowel movements. Physical examination is consistent with evidence of recent weight loss and abdominal examination is normal. A stool specimen is guaiac-negative. There are scattered ecchymoses on all 4 extremities. Laboratory results reveal an albumin of 2.9 g/dL (normal >3.8 g/dL), INR 1.9 (normal 1.0 in patients who are not taking warfarin), and normal liver function tests. A trial of a gluten-free diet is attempted. There is no change in symptoms over the subsequent 3 weeks, and she has an additional 4-pound weight loss. Stool cultures for enteric pathogens and ova and parasites are negative on 3 occasions. Which of the following is the most likely diagnosis? A. Celiac sprue B. Enterohemorrhagic Escherichia coli C. Enterotoxigenic Escherichia coli D. Pancreatic insufficiency E. Tropical sprue

E. Tropical sprue This patient presents with signs of malabsorption as described by her loss of weight, frequent loose stools, and evidence of multiple vitamin deficiency. She is also hypoalbuminemic and has just returned from a tropical region. There are no risk factors or evidence, by history, for liver disease. The hypoalbuminemia, ecchymosis, and INR are explained by the malabsorptive process instead, which is consistent with a vitamin K deficiency. The findings are consistent with a tropical sprue. These patients often develop a vitamin B12 deficiency, as the terminal ileum is also affected more severely in the proximal small bowel.

A 61-year-old man presents with colicky abdominal pain and vomiting of 3 days duration. On physical examination there is moderate distention, high-pitched hyperactive bowel sounds, and a 5-cm tender groin mass. On direct questioning he explains that he has "had that bulge for many years." He has always been able to "push it back in" when he lies down but for the past 3 days he has been unable to do so. His temperature is 38.9ºC (102ºF) and white blood cell count 12,500/mm3 (normal 5,000-10,000/mm3). Which of the following is the most appropriate management at this time? A. Sonogram of the mass B. Trial of nasogastric suction and IV fluids for a few days C. Insertion of a long rectal tube via sigmoidoscopy D. Manual reduction of the hernia, followed by a period of observation E. Urgent surgical intervention

E. Urgent surgical intervention The clinical picture is that of a strangulated or irreducible inguinal hernia. If he only had the tender mass without signs of intestinal obstruction, he might have omentum trapped. If he had no fever with a normal white blood cell count but a tender mass, he could be obstructed but without strangulation. The combination that he has, however, is clearly that of obstruction with strangulation. He needs urgent surgery. Delay in surgery can lead to necrotic bowel, which increases both morbidity and mortality.

A 50-year-old chronic alcoholic presents with dementia, paralysis of lateral gaze, and difficulty walking. Which of the following vitamin deficiencies is most likely responsible for this clinical picture? A. Vitamin B12 B. Vitamin B6 C. Folate D. Vitamin D E. Vitamin B1 (thiamine)

E. Vitamin B1 (thiamine) This patient has all the signs of Wernicke's encephalopathy, a disease characterized by dementia, ataxia, and ophthalmoplegia. This disease typically occurs in alcoholic patients and is the result of thiamine deficiency. Risk factors for thiamine deficiency include alcohol dependence, malabsorption, and a diet low in thiamine, such as the consumption of polished rice or an alcoholic who obtains the majority of his calories via alcohol rather than food. Wet beriberi can also be a complication of thiamine deficiency, which can present with high-output heart failure with edema and orthopnea. Dry beriberi, another complication, occurs with chronic deficiency states and is associated with distal peripheral polyneuropathy.

120. A 45-year-old woman has been experiencing generalized weakness and a sensation of "pins and needles" for the past 3 weeks. She exercises daily, rarely drinks alcohol, and is a strict vegetarian. Her temperature is 37ºC (98.6ºF), blood pressure 110/70 mm Hg, pulse 60/min, and respirations 18/min. Examination shows weakness of the proximal and distal muscles of the lower extremities. Deep tendon reflexes are increased. The gait is ataxic. Which of the following is the most likely cause of these symptoms? A. Guillain-Barré syndrome B. Lambert-Eaton syndrome C. Myasthenia gravis D. Polymyositis E. Vitamin B12 deficiency

E. Vitamin B12 deficiency The correct answer is E. This patient has subacute combined degeneration of the spinal cord, which is caused by a vitamin B12 deficiency. It is most often caused by pernicious anemia, but it may be acquired by patients who have strict vegetarian diets or small bowel disease. The clinical manifestations include weakness, paresthesias, loss of vibratory sensation, increased deep tendon reflexes, and extensor plantar responses. The gait is ataxic. Mental changes may also occur. The diagnosis is made by measuring serum vitamin B12 levels, homocysteine, and methylmalonic acid. Vitamin B12 deficiency occurs only after years of being a vegetarian, because vitamin B12 stores are present for years before depletion occurs. Neurologic changes caused by pernicious anemia are considered to be irreversible, so early treatment is warranted. The treatment is vitamin B12 replacement

A 5-day-old full-term boy born at home is being evaluated in the urgent care clinic for bruising and gastrointestinal bleeding. Laboratory findings include: Partial thromboplastin time and prothrombin time >2 minutes (normal for newborns ≤60 seconds) Serum bilirubin 4.7 mg/dL (normal for newborns) Prothrombin time 15.2 seconds (normal 11-13 seconds) Alanine aminotransferase 18 mg/dL (normal <45 mg/dL) Platelet count 330,000/mm3 (normal 150,000-300,000/mm3) Hemoglobin 16.3 g/dL (normal 13.2-21.0 g/dL) The boy's mother has factor V Leiden deficiency. Which of the following is the most likely cause of the boy's bleeding? A. Factor VIII deficiency B. Factor IX deficiency C. Idiopathic thrombocytopenic purpura D. Liver disease E. Vitamin K deficiency

E. Vitamin K deficiency The infant in this clinical vignette has hemorrhagic disease of the newborn as a result of vitamin K deficiency. This was a major cause of bleeding in neonates in the past, but it is now uncommon because of the routine administration of vitamin K at birth. It is still encountered in situations in which infants are born outside the hospital, however. The normal newborn has a moderate deficiency of the vitamin K-dependent coagulation factors. The plasma levels of these factors fall even further during the first 2-5 days of life, rise again when the infant is 7-14 days old, and attain normal adult levels at about 3 months of age. This variation usually does not produce any bleeding or bruises. In hemorrhagic disease of the newborn, however, the initial fall is accentuated, and the restoration is delayed and incomplete. As a result, coagulation abnormalities become severe and bleeding may occur. In order to prevent this deficiency state with its subsequent bleeding, all newborns should receive 0.5-1.0 mg of vitamin K intramuscularly within the first hour after birth.

Gallbladder disease is prevalent, and accurate diagnosis is needed in for removal of the diseased gland. Most acute episodes of acute cholecystitis are due to symptomatic gallstones becoming lodged in the cystic duct, causing obstruction and preventing the gallbladder from emptying. Risk factors for the development of gallstones include all of the following EXCEPT: A. Ileal disease B. Pregnancy C. Hyperlipidemia D. Female sex E. Weight loss

E. Weight loss Weight loss results in lack of stimulation of gallbladder contraction, which leads to concentrated bile that can provoke episodes of gallbladder colic. It is not weight loss that leads to gallstone formation, but rather weight loss that leads to lack of gallbladder stimulation, provoking gallbladder colic symptoms with the potential for acute cholecystitis occurring.

A teenage patient is seen in the emergency department complaining of right lower quadrant pain. Several teenagers in the community have had similar complaints. Which of the following causes of infectious enterocolitis has a predilection for the terminal ileum and may cause an erroneous diagnosis of acute appendicitis? A. Shigella B. Salmonella C. Campylobacter D. E. coli 0157:H7 E. Yersinia

E. Yersinia Yersinia enterocolitis can cause acute ileitis and is a cause of the so-called "infectious appendicitis," as patients will complain of pain in the right lower quadrant. Yersinia has a predilection for the terminal ileum, and inflammation in this area will lead to symptoms that are similar to those seen with appendicitis.


संबंधित स्टडी सेट्स

Sociology - Chapter 7: Deviance, Crime, and Social Control (self quiz; multiple choice)

View Set

APCSP Chapter 2: Check your understanding

View Set

Workforce Safety & Wellness / Lifting & Moving Patients (CH 2 & CH 8)

View Set

A&P Chapter 8 Practice Questions

View Set

Health and Life Comprehensive Exam

View Set

Behavioral Statistics (Final Exam)

View Set

Unit 2: Module 1: MANAGEMENT INFORMATION SYSTEMS AND BUSINESS FUNCTIONS

View Set